[ /tv/ /rf/ /vg/ /a/ /b/ /u/ /bo/ /fur/ /to/ /dt/ /cp/ /oe/ /bg/ /ve/ /r/ /mad/ /d/ /mu/ /cr/ /di/ /sw/ /hr/ /wh/ /lor/ /s/ /hau/ /slow/ /gf/ /vn/ /w/ /ma/ /azu/ /wn/ ] [ Main | Settings | Bookmarks | Music Player ]

No.136767 Reply
File: valley_of_the_dark_magi_by_jesselindsay.jpg
Jpg, 134.81 KB, 600×713 - Click the image to expand
edit Find source with google Find source with iqdb
valley_of_the_dark_magi_by_jesselindsay.jpg
Наша кафедра продолжает свою работу. Здесь мы обсуждаем математику, а также иногда отвечаем на простые вопросы доброанонов.
Тред обучения математике: >>124265

Библиотека: http://libgen.org/

Калькуляторы:
http://online-integral.ru/
http://www.wolframalpha.com/

Предыдущий: >>133509
>> No.136770 Reply
>>136765
Ок. Прочитай для начала трёхтомник Кострикина по алгебре.
>> No.136787 Reply
Ряд тейлора для некоторых функций вроде e^x верен не для окрестности точки вокруг которой смотрим, а по всему R из-за того что функция бесконечно много раз дифференцируема и бесконечно непрерывна?
Продублирую
>> No.136788 Reply
>>136787
Нет. Есть бесконечно много раз дифференцируемые функции, у которых ряд Тейлора не сходится к функции ни в какой окрестности никакой точки (не аналитические):
enwiki://Non-analytic_smooth_function#A_smooth_function_which_is_nowhe[...]lytic
enwiki://Fabius_function
и т.д. очень много интересных примеров

В комплексной плоскости такого не может быть: если функция хотя бы один раз (комплексно) дифференцируема в окрестности точки, то она бесконечно дифференцируема и аналитична в окрестности точки (представляется рядом Тейлора). Тут важна именно дифференцируемость в окрестности, а не только в самой точке.
>> No.136789 Reply
>>136788
Так чому если ряд Тейлора в определени имеет только некоторую окрестность мы имеем сходящиеся ряды для любого x и формулы общего вида синусов/косинусов/e^x. Алсо, правильно ли я понимаю, что непрерывность в точке=предел функции в точке=дифференциируемость в точке?
>> No.136790 Reply
>>136789
Ну простой ответ: повезло. Ряд Тейлора аналитической функции сходится к самой функции внутри круга сходимости. У экспоненты (а синус и косинус - это, по сути, линейные комбинации комплексных экспонент) коэффициенты ряда Тейлора стремятся к 0 так быстро, что радиус сходимости бесконечен.
> Алсо, правильно ли я понимаю, что непрерывность в точке=предел функции в точке=дифференциируемость в точке?
Нет, м.б. ты что-то другое имел в виду?
непрерывность в точке=предел функции в точке равен значению функции в точке
дифференциируемость в точке=функция в окрестности точки приближается линейной; для функций одной переменной это то же самое, что существование производной в точке (линейный коэффициент как раз будет равен производной)
>> No.136791 Reply
>>136790
Я подразумевал, что определение непрерывности = определению существования предела функции в точке = определению дифференциируемости/существованию производной в точке же.
>> No.136792 Reply
>>136791
Нет, это совершенно разные вещи. У функции может быть предел в точке с обоих сторон, а значение в точке будет другим и тогда функция не будет непрерывной. Функция может быть непрерывной и иметь все пределы, но не иметь производной - например, |x| в нуле.
>> No.136793 Reply
>>136792
Я уже понял. Нет, это одни и те же определения, бака. Функция непрерывна в точке если левосторонний предел и правосторонний предел есть и они равны или что левостороння производная = правосторонней производной. И наоборот. Шах и мат, аметист
>> No.136799 Reply
>>136793
Что ты имеешь в виду под "одними и теми же определениями"? Сходство между определениями есть, конечно, но по смыслу они совершенно разные, так как определяют разные объекты.
>> No.136862 Reply
>>136767
Анон, какой есть годный задачник по абстрактной алгебре? Хочу порешать задачи на отображения - морфизмы между структурами, ибо плохо ориентируюсь в этой теме.
>> No.136864 Reply
>>136862
Приведи пример такой задачи.
>> No.136865 Reply
>>136862
Начни с "тривиума по алгебре" Каледина.
>> No.136874 Reply
Где можно почитать про интегралы, которые зависят от параметра? Для чего вообще они нужны?
>> No.137008 Reply
>>136767
как представить дифференциальное уравнение в конечно-разностной форме
>> No.137012 Reply
Как перестать делать ошибки в расчётах? Всё время на этом лажаю. То случайно вместо плюса минус напишу, то корень неправильно извлеку. Как прокачать эту математическую внимательность и пофиксить рассеянность, в общем?
>> No.137035 Reply
>>137012
> случайно вместо плюса минус напишу
попытаться посчитать какую-нибудь физически-важную фигню, и представить себе масштабы последствий воплощения в жизнь идеи, основанной на неверных расчетах
> корень неправильно извлеку
извлекай правильно же :3
>> No.137036 Reply
>>137035
> представить себе масштабы последствий воплощения в жизнь идеи, основанной на неверных расчетах
ссылку на мультик ему надо, где полтора землекопа и всё такое.
>> No.137110 Reply
File: 10202.jpg
Jpg, 104.53 KB, 300×418 - Click the image to expand
edit Find source with google Find source with iqdb
10202.jpg
МОжет у кого есть в бумажном виде в дс? Очень надо, я бы выкупил у вас.
>> No.137111 Reply
>>137110
Зачем надо?
>> No.137113 Reply
>>137012
Проверяй результат, и если что-то не так, переделывай.
>> No.137166 Reply
>>137111
Это очень хорошая книжка, просил товарищ из Литвы. Он уже как второй год пытается искать по объявлениям и даже в издательство писал, но там нет ее :C
>> No.137171 Reply
Где можно прочитать в доступной форме про кратные интегралы?
>> No.137217 Reply
File: csi-gil-grissom.jpg
Jpg, 18.04 KB, 500×319 - Click the image to expand
edit Find source with google Find source with iqdb
csi-gil-grissom.jpg
>>137171
погугли решения задач Волькенштейна по физике, должен какой-то сайт на народе вылезти, на котором раздел по матану есть, там годные пдф'ки лежат. именной какой то сайт вроде.
>> No.137218 Reply
>>136767
Анон, посоветуй задачник по линейной алгебре для глубокого проникновения в теорию. Что-то вроде задачников по теории групп. Технические упражнения на вычисление скучны.
>> No.137290 Reply
>>137218
Попробуй
Прасолов - Задачи и теоремы линейной алгебры
ftp://ftp.mccme.ru/users/prasolov/linalg/linalg.pdf
>> No.137291 Reply
>> No.137300 Reply
Имеет ли смысл читать старые выпуски «Кванта», или лучше читать книги «Библиотечки "Кванта"»?
>> No.137304 Reply
>>136767
Анон, напомни чоткий, ровный, пацанский признак подполя. На первом курсе нам его давали, а в интернете я не могу найти эту теорему.
>> No.137306 Reply
>>137304
Замкнуто относительно вычитания и деления, содержит 0 и 1.
>> No.137307 Reply
>>137306
А, то есть
4) Произведение x и обратного к x лежит в подмножестве
2) Разность двух элементов из подмножества лежит в нем же
Тогда это подполе?
Отсюда же вытекает наличие 0 и 1: x - x, x/x
>> No.137310 Reply
>>137307
> вытекает
Не вытекает, потому что замкнутое относительно этих операций подмножество может быть пустым. Чтобы отсечь эту патологию, нужно явно требовать непустоту. Поле не может быть пустым множеством.
>> No.137351 Reply
>>136767
Анон, где можно скачать бесплатно "Упражнения по группам, кольцам и полям: учебное пособие"?
>> No.137358 Reply
>>137310
А зачем подполе ваще введено?
>> No.137424 Reply
Кто-нибудь, запилите перекат для физика-треда >>113576, а то он уже давно в бамплимите.
>> No.137426 Reply
>>137424
Запилил.
>> No.137466 Reply
Как по-русски будет «annihilated by» в контексте «The hyperplane (i.e. subspace of codimension 1) annihilated by a covector f∗ ∈ V∗ is the set of all vectors x ∈ V such that f∗(x) = 0»?
>> No.137467 Reply
>>137466
Никак. Обычно это переводят как "сокращаются" или "взаимно уничтожаются", но здесь это не очень уместно.
>> No.137468 Reply
>>137467
Тем не менее, я сомневаюсь, что задание гиперплоскости посредством ковектора нигде не встречается в русскоязычной литературе.
>> No.137471 Reply
>> No.137472 Reply
>>137466
Мне кажется, в русской литературе ковекторы как-то меньше употребяются.
Обычно говорят "линейный функционал". Соответственно, вектор его обнуляет, принадлежит его ядру и т.д.
>> No.137474 Reply
>>137472
Ядро линейной формы
>> No.137527 Reply
>>137466
Частенько это передают словом "ортогональность". Гиперплоскость, ортогональная ковектору f*(Впрочем, ковекора в русскоязычной литературе любят называть линейными функционалами или даже двойственными векторами).
>> No.137534 Reply
>>137527
Это вообще говоря правильно только в случае, когда в пространстве есть метрика. Иначе никакой ортогональности не бывает. Но как жаргон - да, прокатит.
>> No.137538 Reply
>>137534
Что ты имеешь в виду под "правильным"?. Если уж буквоедствовать, то для ортогональности в строгом смысле этого слова и метрики недостаточно, а нужна ещё и полнота.
>> No.137540 Reply
>>137538
Почему полнота? Мне казалось, ортогональные вектора, это такие что их скалярное произведение ноль. А для подпространств - произведение любых двух векторов из них.
>> No.137541 Reply
>>137540
Если у тебя двойственное пространство V* не вкладывается в V, то никакого скалярного произведения вектора и ковектора просто не существует - как ты будешь перемножать вектора, лежащие в разных пространствах? В конечномерном случае, конечно, V* c V изоморфны и перемножить можно, но, вообще говоря, в бесконечномерном случае для того, чтобы V* совпадало с V и существовало скалярное произведение, нужно, чтобы V было полно как топологическое пространство.
>> No.137554 Reply
Какая размерность ядра и размерность образа отображения тут? Мне просто, чтоб проверить и убедится.
   1 1 1
   0 1 1
-1 0 0
>> No.137555 Reply
>>137554
http://www.wolframalpha.com/input/?i=null+space+of+[[1%2C1%2C1]%2C[0%2[...]2C0]]
Размерность ядра - 1,
Размерность образа соответственно 2.
>> No.137556 Reply
File: tumblr_ni8ajwqrf21s076u4o1_1280.png
Png, 1522.51 KB, 1280×1658 - Click the image to expand
edit Find source with google Find source with iqdb
tumblr_ni8ajwqrf21s076u4o1_1280.png
>>137555
Псиба. Все отлично.
>> No.137571 Reply
Доброанон, помоги, пожалуйста. А то туплю чего-то.

enwiki://Banach_fixed-point_theorem
Зачем для доказательства понадобилось лемма 2? Почему нельзя было сразу из первой леммы перейти к пределу (ведь q^n ушло бы в ноль)?
>> No.137575 Reply
>>137571
К какому пределу, собственно? Из первой леммы мы получаем только, что расстояние между n-ным и n+1-ым членом последовательности равно константе, умноженной на q^n. Существование предела ещё ниоткуда не следует, его-то и доказывает вторая лемма. Более того, даже если бы мы знали, что некий предел х существует, то перейдя к пределу по n в первой лемме, мы бы получили попросту d(x,x)=0, что и так очевидно.
>> No.137576 Reply
>>136767
Учился на пме в свое время 2 курса, потом пришлось пойти на вечернее, я немного офигел от контингента там, он не просто тупой, а ТУПОЙ и гыгыкающий, решил пойти не лекцию по теорверу, думаю ну будет интересно, тем более нам препод в свое время говорил мол сложно и интересно. Пришел на лекцию и офигеваю, препод тупо показывает слайду и тупо читает что на них написано, ВСЕ, больше ничего. Так потом я еще узнал что это приглашенный преподаватель, левый какой-то. Мне обидно что я потерял свой пм, но что было то было, но тут качество ужас.
Вот примеры лекции. http://rghost.ru/8Mrm2XLkm
Подскажите хороший учебный, не сильно перегруженный теорией, с хорошим количеством практики.
>> No.137579 Reply
>>137575
> Более того, даже если бы мы знали, что некий предел х существует, то перейдя к пределу по n в первой лемме, мы бы получили попросту d(x,x)=0
Мы же определили x.{n+1}=f(x.n), получили бы d(f(x.n),x.n)->0, не?
>> No.137580 Reply
>>137579
> d(f(x.n),x.n)->0
Ну да. Только f(x.n) стремится к х, и х.n тоже стремится к х. Вот и получается d(x,x)=0 при переходе к пределу.
>> No.137581 Reply
>>137579
Кажется, ты думаешь, что если d(x.(n+1),x.n)->0, то последовательность {x.n} сходится. Так вот, это неверно, иначе бы сходился гармонический ряд.
>> No.137583 Reply
>>137580
> Только f(x.n) стремится к х
Но при этом f(x.n)->f(x), то есть получаем f(x)=x, что и требовалось доказать. То есть поучается, что лемма 2 нужна была только для доказательства существования предела, так?
>> No.137584 Reply
>>137583
> То есть поучается, что лемма 2 нужна была только для доказательства существования предела, так?
Ну да.
>> No.137585 Reply
>>137584
Спасибо, прояснилось.
>> No.137601 Reply
Как выражается свёртка тензора в безиндексной нотации?
>> No.137603 Reply
>>137601
Tr[...]
>> No.137609 Reply
>>137603
Это для тензоров вида V⊗V*, а что насчёт насчёт тензоров более высоких рангов?
>> No.137613 Reply
>>137609
А как ты в безындексной нотации покажешь, в каких пространствах сворачивать? Иногда это ясно из контекста, или можно у Tr снизу что-нибудь пририсовать, чтоб было понятно. По крайней мере я ничего другого не видел, или не помню.
>> No.137634 Reply
File: tv_nsu07.pdf
Pdf, 1.21 KB, 595×842 - Click the image to get file
tv_nsu07.pdf
>>137576
> не сильно перегруженный теорией, с хорошим количеством практики
На русском языке такого мало, увы

Советуют Феллер - Введение в теорию вероятностей и её приложения, но я сам не читал
Ещё приложил к посту лекции, вроде неплохие, когда-то готовился по ним
>> No.137648 Reply
>>137634
Cпасибо, а семинар по дискретке вообще "пиздец" был, 2 часа потратить чтобы объяснять что такое перестановка на примере abc acb и т.д., а также количество комбинаци на примере 0001 и т.д. еще лошадок рисовать не хватало для детей, у нас раньше за 15 это все объясняли, какой ад. Хорошо что существует андерсон.
капча: достойно смеха
>> No.137649 Reply
>>137634
Cпасибо, а семинар по дискретке вообще "пиздец" был, 2 часа потратить чтобы объяснять что такое перестановка на примере abc acb и т.д., а также количество комбинаци на примере 0001 и т.д. еще лошадок рисовать не хватало для детей, у нас раньше за 15 это все объясняли, какой ад. Хорошо что существует андерсон.
капча: достойно смеха
>> No.137674 Reply
>>137648
>>137649
Советую учить английский. На западе реально учебники для прикладников гораздо лучше: с примерами, картинками и т.д.
Плюс много видеолекций и других материалов в интернете.
Вот, например, какой-то курс: https://www.youtube.com/playlist?list=PLUl4u3cNGP60A3XMwZ5sep719_nh95qOe (английские субтитры)
>> No.137749 Reply
>>137674

Жаль, что торрентами их лекции нигде не выложены. Качать с ютубов и вебархивов не очень-то удобно.
>> No.137847 Reply
Помогите разобраться с соотношением.
Вопрос по учебнику Вьюгина "Математические основы теории машинного обучения и прогнозирования" (http://www.iitp.ru/upload/publications/6256/vyugin1.pdf).
На странице 38 есть неравенство, помеченное "(1.15)". Никак не могу понять, на основании какого свойства биномиального распределения получили 1/2 в правой части неравенства.
>> No.137880 Reply
>>137749
Youtube-dl умеет скачивать весь плейлист целиком.
>> No.137897 Reply
>>137847

Там же дальше объясняется, начиная со слов «Действительно»
>> No.137900 Reply
У меня психологический вопрос. Я знаю, что я просто тупой, а у математиков больше серого вещества, чем у меня, и они имеют больше оперативки, они умные. Но всегда было интересно, как они субъективно воспринимают числа? Я вот понял, что их с детства проговариваю как песенку, «дважды два читыре пятью семь трицатьпять», поэтому не могу ничего считать в уме: слова слишком длинные и вытесняют друг друга. Если постараюсь визуализировать (причём что – саму цифровую запись, а не какие-нибудь, например, мощности точек или числовые понятия как таковые), то с трудом умножу 3-значное число на двухзначное, ну может быть с огромными затратами времени – 3-значное на 3-значное. Это же пиздец.
А если визуализировать, например, точки и степени двойки – до 7-8-9 точек легко на плоскости, 8 это 2 квадрата или вершины одного куба, 16 это куб плюс его смещение, 64 это куб с вершинами из кубов (смутно могу увидеть), а вот 512 – куб из кубов кубов или куб со стороной 8 – уже никак не вижу. Если бы я мог пойти дальше 3Д, но...
И ведь это всего лишь степени двойки. А дробные числа, а дробные степени, как это вообще мыслить? А какие-нибудь хайлевельные операции, типа дифференциирования? А чисто математические понятия, все эти ваши окрестности, алгебры, ряды?

Матанобоги, так как вы мыслите числа и операции над ними? Что такое число?
>> No.137902 Reply
>>137900
Ты программировать умеешь? Классы, объекты, циклы воображаешь как-то?
>> No.137903 Reply
>>137900
Математика — не числодрочество, а абстракции и отношения между ними.
> А дробные числа, а дробные степени, как это вообще мыслить?
Использовать навык под названием «абстрактное мышление».
> Что такое число?
Математический объект, который обычно связывают с подсчётом и измерением.
>> No.137904 Reply
>>137902
Нет, я не умею программировать.
>>137903
Это не ответ. Человеческий мозг приспособлен к обработке сенсорной информации. Как ощущается твоё абстрактное мышление? У тебя вообще есть рефлексия?
>> No.137907 Reply
>>137904
> Человеческий мозг приспособлен к обработке сенсорной информации.
Но умеет далеко не только это, спасибо длительной эволюции с развитием речи.
> У тебя вообще есть рефлексия?
Как ты визуализируешь рефлексию?
> Как ощущается твоё абстрактное мышление?
Смотри, ты ведь можешь использовать слово «пиздец» без визуализации конкретных видов пиздецов? Как и слово «предмет», например.
Точно так же и числами и множествами.

А вообще, тебе в >>39559.
>> No.137927 Reply
File: 2015-03-11_14-10-30.png
Png, 1.31 KB, 158×44 - Click the image to expand
edit Find source with google Find source with iqdb
2015-03-11_14-10-30.png
Как продифференцировать пикрилейтед?
>> No.137930 Reply
>> No.137931 Reply
>>137930
Я это видал, но если под интегралом есть функция еще и от икс, то я затрудняюсь. Что меняется, если там икс?
>> No.137934 Reply
>>137927 возьми его, возьми его частями! опутывай его пределами интегрирования, пока он не превратится в константу и будет лишь жалобно всхипывать под твоим натиском. дифференцируй его нежно, и по икс, и по игрек, как будет угодно!
>>137931 а чего тебе икс-то, коль суммируешь по тэ? он сидит себе смирно и глазками хлопает
>> No.137936 Reply
>>137934
Ок ок, пасиба, вроде понял, в какую сторону делать.
>> No.137949 Reply
>>137934
> а чего тебе икс-то, коль суммируешь по тэ? он сидит себе смирно и глазками хлопает
Ты мне препода мне напоминаешь какого-то, но я не помню какого.
>> No.138032 Reply
File: 1378980121504.jpg
Jpg, 89.54 KB, 604×453 - Click the image to expand
edit Find source with google Find source with iqdb
1378980121504.jpg
Всех с днем пи, пацаны.
>> No.138033 Reply
>>138032
И тебя, анон.
>> No.138046 Reply
File: 1426352292909.png
Png, 0.96 KB, 300×20 - Click the image to expand
edit Find source with google Find source with iqdb
1426352292909.png
>>137900
Не математик.
Складываю натуральные числа. Знаю суммы чисел до 9, дальше по разрядам. Легко, когда сагаемое кратно десяти.
Если нужно складывать в другой системе счисления, то перевожу через десятичную систему.
>> No.138047 Reply
>>137900
Ещё. Может я не о том написал.
Тут типа api разрабатыватеся для образов, с которыми сталкиваешься. Если знать анализ с действительными числами, потом можно к топологии прийти. Ну так и получилось.
>> No.138060 Reply
Как вычисляется объём шара? А объём конуса? Формулы-то нам дали, но как они выводятся у этих округлых фигур? Площадь шара - как выводится?
мимоиз11_класса. Я подозреваю, что всё это доказывается через какой-нибудь анализ и разбиение на маленькие части, как у Архимеда. Но учусь только по школьной программе, объясните, пожалуйста, попроще.
>> No.138063 Reply
>>138060
Все разбивается на круги, площадь которых выражается функцией, и интегралом суммируется по соответствующим пределам. Погугли тему определенный интеграл, там даже есть такое упражнение "посчитайте объем фигуры вращения".
>> No.138065 Reply
File: images-(7).jpg
Jpg, 5.23 KB, 229×140
edit Find source with google Find source with iqdb
images-(7).jpg
File: cd-cases-oblique.jpg
Jpg, 72.54 KB, 800×600
edit Find source with google Find source with iqdb
cd-cases-oblique.jpg

>>138060
ruwiki://Мера_множества
На основе теории меры определяется интеграл. Интеграл - штука, позволяющая измерять множества. Ты определяешь шар, конус, окружность и прочее как множество точек r трёхмерного пространства таких, что f(r) = 0 для некоторой функции f. Чтобы получить площадь или объём, нужно эту функцию f соответствующим образом проинтегрировать. Конкретные методы можно нагуглить по словам "объём тела вращения", "определённый интеграл".

Можно говорить более абстрактно и ввести так называемую характеристическую функцию χ(r) геометрической фигуры. Характеристическая функция χ в точке r равна единице, если точка r принадлежит фигуре, в противном случае характеристическая функция равна нулю. Интеграл характеристической функции и будет объёмом (либо площадью, длиной) геометрической фигуры. Чтобы считать объёмы, достаточно научиться интегрировать характеристические функции. Довольно большой класс характеристических функций можно интегрировать с помощью интеграла Лебега. Построение теории интеграла Лебега встречается во многих современных учебниках. В более старых учебниках пользуются менее мощным интегралом Римана.

ruwiki://Метод_исчерпывания
ruwiki://Принцип_Кавальери
ruwiki://Определённый_интеграл
ruwiki://Интеграл
ruwiki://Площадь_круга
ruwiki://Тела_вращения
ruwiki://Поверхность_вращения
>> No.138067 Reply
>>138060
> Я подозреваю, что всё это доказывается через какой-нибудь анализ и разбиение на маленькие части
Правильно подозреваешь - площадь сечения интегрируется по высоте. Доказательство "через разбиение" - по сути, то же интегрирование, только с прямым выписыванием интегральной суммы и переходом к пределу. Других доказательств я не встречал. Не понимаю, чем тебя не устраивают эти - насколько я помню, интегралы и пределы есть в школьной программе.
>> No.138085 Reply
И так,мне требуется помощь.Задача следующая:
Найти все 4х значные числа,квадраты которых кончаются на эти же числа.

Ответ найден,это 9376

Осталось доказать единственность,или опровергнуть это и найти оставшиеся числа. 9376 я нашел проверяя то,какие числа в квадрате на что заканчиваются. например 1,5,6 в квадрат заканчиваются на те же цифры,из двухзначных только 25 и 76, если смотреть трехзначные,то они явно должны заканчиваться либо на 25 либо на 76 (тут то и начинается проблема,ибо я не уверен,не могу доказать единственность),получаем что из трехзначных у нас только 625 и 376 в квадратах дают то,что нам надо. Ну,и перебором из 18 вариантов находим 9376.

У меня была мысль,что число можно представить через a^2 - a ⋮10000.Но че дальше с этим делать - х знает,то ли сказать,что а(а-1)⋮10000,а потом 10000 разожить на простые ((2^4)*(5^2) ) и как-то по малой теореме ферма,то ли че. ПОмогите пожалуйста
>> No.138088 Reply
>>138085
Докажи полным перебором, это сейчас нетрудно.
http://ideone.com/COX3hb
>> No.138090 Reply
>>138088
Написать программу изи,но в том-то и дело,что надо без этого,программный код не будет считаться доказательством.
>> No.138092 Reply
>>138090
По какой причине не будет?
>> No.138093 Reply
>>138085
Пусть а - искомое четырёхзначное число. Имеем а^2 = a(mod 10000). По китайской теореме об остатках это означает, что а^2 = a(mod 16) и а^2 = a(mod 625)(потому что 16*625 = 10 000 и 16 с 635 взаимно просты). Первое сравнение решаем перебором, второе - усложнённым перебором(т.е решаем сначала а^2 = a(mod 5), затем а^2 = a(mod 25) с остатком по модулю 5, полученным из предыдущего сравнения, потом а^2 = a(mod 125) с известным остатком mod 25, ну и а^2 = a(mod 625) с известным остатком mod 125).
>> No.138094 Reply
>>138093
> 16 с 635 взаимно просты
С 625, конечно.
>> No.138095 Reply
>>138085
Придумал более элегантный вариант: а(а-1)⋮10000. При этом числа а и а-1 отличаются на единицу, а значит у них нет ни одного общего простого делителя. А значит, либо а делится на 16 и (а-1) на 625, либо наоборот.
>> No.138102 Reply
File: IMG_20150227_2140...
Jpg, 1751.45 KB, 2448×3264
edit Find source with google Find source with iqdb
IMG_20150227_214021.jpg
File: IMG_20150227_2138...
Jpg, 1464.78 KB, 2448×3264
edit Find source with google Find source with iqdb
IMG_20150227_213857.jpg
File: IMG_20150227_2139...
Jpg, 1391.37 KB, 2448×3264
edit Find source with google Find source with iqdb
IMG_20150227_213927.jpg
File: IMG_20150227_2138...
Jpg, 1454.11 KB, 2448×3264
edit Find source with google Find source with iqdb
IMG_20150227_213843.jpg

>>137634
Спасибо господа, проболел сильно лекции и работу. Теперь вот надо задания сделать, спрошу глупые вопросы, не пинайте, ладно?
1) 5.3 задание - Тут надо найти по формуле независимых испытаний где р - вероятность попадания? и от р уже смотрим попал в интервал или нет. верно?
2)5.4 - Тут просто производную что ли взять? В чем смысл, слишком просто же
3) 5.6 немного не понял что гуглить, подскажите по каким тегам.
4)5.1 и 5.2 - ньютона лейбница?
>> No.138204 Reply
>>136767
няши, объясните пожалуйста понятно что такое линейная оболочка системы векторов?
>> No.138205 Reply
>>138204
>>131437 и ниже
>> No.138285 Reply
File: Без-имени20150116001956.png
Png, 615.75 KB, 1184×849 - Click the image to expand
edit Find source with google Find source with iqdb
Без-имени20150116001956.png
Не знаю где просить помощи, спрошу здесь. Есть 3 примера:
f(x1x2x3)= 10011010 - 3 способа
f(x1x2x3x4)= v (0,2,4,6,7,8,10,14)
f(x1x2x3x4)= ^ (2,7,6,11,13,14,15)
срочно нужно решить их до понедельника, сам не в ладах с дискретной математикой и вообще даже тему не знаю, помогите пожалуйста, хотяб узнать что за тема и что сделать надо. Вы моя последняя надежда..
>> No.138297 Reply
>>138285
Тема - булевы функции. С ними нужно что-то сделать.
>> No.138299 Reply
File: e80c206b.jpg
Jpg, 73.13 KB, 640×480 - Click the image to expand
edit Find source with google Find source with iqdb
e80c206b.jpg
>>138285
Я не понял.
>> No.138306 Reply
>>138297
Спасибо, буду знать что в поиске вбивать
>> No.138386 Reply
Здесь сидел няша воннаби-математик. Я с ним списался на сосаче. Няша внезапно удалил обе страницы и не выходит на связь. Я волнуюсь, отзовись! Надеюсь ты просто заперся и ботаешь, а не выпилился.
>> No.138387 Reply
>>138386
Их было несколько. Ты про которого?
>> No.138393 Reply
File: Anime-Original-Anime-Art-Anime-1653043.jpeg
Jpeg, 599.96 KB, 2179×1234 - Click the image to expand
edit Find source with google Find source with iqdb
Anime-Original-Anime-Art-Anime-1653043.jpeg
Заранее прошу прощения, если вопрос платиновый.
Я студент-биолог, хочу нормально изучить статистику (а не то, что нам давали на парах под ее видом). Для этого мне нужен хороший учебник. Математика мне дается тяжело, так что желательно, чтобы он был на русском и подробно разжевывал ключевые вопросы, а если с примерами и задачками, то вообще великолепно.
>> No.138447 Reply
File: 2015-03-22_17-48-...
Jpg, 2.86 KB, 180×66
edit Find source with google Find source with iqdb
2015-03-22_17-48-16.jpg
File: tumblr_nl9tu9fSo9...
Gif, 1598.80 KB, 500×307
edit Find source with google Find source with iqdb
tumblr_nl9tu9fSo91r922azo1_500.gif

Пацаны, мужской пример. Надо найти все точки разрыва и описать их характер. Точки разрыва очевидно x=j. А вот насчет их рода... Надо доказать что бесконечность перетянет бесконечно малую или наоборот. Помогите хотя бы идеями!
>> No.138452 Reply
>> No.138454 Reply
>>138447
Ох лол. Это выражение не задаёт никакой функции, потому что нигде не сходится. Откуда взят этот пример? Мне интересно посмотреть на учебник с такими ляпами.
>> No.138455 Reply
File: 1426938756493.png
Png, 33.59 KB, 227×223 - Click the image to expand
edit Find source with google Find source with iqdb
1426938756493.png
>>138454
Препод сам скорее всего составлял контрольную. Так что вполне возможно он мог случайно хуйню написать. Бля, походу срсли нигде не сходится...
>> No.138456 Reply
>>138455
..Погоди, а как доказать, что она нигде не сходится? Мб это и есть ответ и все нормально?
>> No.138463 Reply
>>138456
Переписать как сумму логарифмов и оценить снизу суммой логарифмов натуральных чисел от 1 до бесконечности(возможно, выкинув конечное число членов).
>> No.138467 Reply
File: image.jpg
Jpg, 20.47 KB, 236×236 - Click the image to expand
edit Find source with google Find source with iqdb
image.jpg
>>138452
Недостаточно подробно, и учебники я гораздо лучше воспринимаю, чем видеолекции. Но все равно спасибо.
Особо хотелось бы разобраться в выборках, повторностях, мнимых повторностях и оценке достоверности.
>> No.138476 Reply
File: tumblr_nlggxe1Q8g...
Jpg, 51.28 KB, 540×381
edit Find source with google Find source with iqdb
tumblr_nlggxe1Q8g1rl1jado1_540.jpg
File: tumblr_nll7hlSv8y...
Jpg, 52.53 KB, 540×424
edit Find source with google Find source with iqdb
tumblr_nll7hlSv8y1rl1jado1_540.jpg
File: tumblr_nlitamK9Mh...
Jpg, 57.49 KB, 540×405
edit Find source with google Find source with iqdb
tumblr_nlitamK9Mh1rl1jado1_540.jpg

>>138463
Спасибо, анон!
>> No.138518 Reply
Подскажите решение пожалуйста:
Какими свойствами обладает отношение “x делит y” на множестве натуральных чисел?
>> No.138519 Reply
>>138518
Рефлексивно, антисимметрично, транзитивно.
>> No.138522 Reply
>>138519
Благодарю.
>> No.138530 Reply
File: byn.PNG
Png, 12.41 KB, 594×124 - Click the image to expand
edit Find source with google Find source with iqdb
byn.PNG
Как при подстановке переменной степень высчитывается?
>> No.138531 Reply
>>138530
u = sqrt(x).
Значит, u^2 = x.
Значит, x^2 = u^4, а x^3 = u^6.
Но dx нам тоже нужно заменить, на du. du известно, нетрудно посчитать dx.
Будет dx = 2sqrt(x)du = 2udu. Двойку выносим за знак интеграла, u в числителе и знаменателе сокращаем.
>> No.138535 Reply
File: я все понял.jpg
Jpg, 164.85 KB, 400×359 - Click the image to expand
edit Find source with google Find source with iqdb
я все понял.jpg
>>138531
Спасибо.
>> No.138552 Reply
>>138518
Квазиупорядочение, субтрактивная решётка.
>> No.138587 Reply
Доброаноны, нужна книга "Математика: учебно-справочное пособие" Мордкович, Гусев (http://www.ozon.ru/context/detail/id/28302655/). К сожалению, нигде в интернете не могу её найти! Эта ведь не та, или отличается только мягкой обложкой (http://www.ozon.ru/context/detail/id/5052392/)?
>> No.138603 Reply
>>138587
libgen.org
Мордкович Гусев Математика Справочник школьника
>> No.138630 Reply
>>138603
Забыл указать, что нужно печатное издание.
>> No.138637 Reply
File: 1363789797710.jpg
Jpg, 40.30 KB, 700×553 - Click the image to expand
edit Find source with google Find source with iqdb
1363789797710.jpg
Математики, помогите разобраться с >>137423.
>> No.138787 Reply
Поясните школьнику, как решить задачу на нахождение наибольшего объёма правильной четырёхугольной пирамиды с боковым ребром 6*скврт3
Я беру апофему за х, вывожу из неё грань основания, а из неё и апофему - высоту. В ответе, объём задаётся рациональным выражением, а высота - иррациональным, в результате чего при вычислении производной выходит каша. Вот как я считал:
апофема=х
грань основания=скврт(108-х^2)*2
высота=скврт(х^2-108+x^2)=скврт(2х^2-108)
объём=1/3x^2скврт(2х^2-108)
При подсчёте производной выходит дикая каша, а это очевидно не то, что нужно. Как подсчитать так, чтобы высота была рациональной?
>> No.138789 Reply
а, всё решил уже, надо былос высоту за икс взять
>> No.138793 Reply
File: 20150329_183632.jpg
Jpg, 2045.20 KB, 3264×2448
edit Find source with google Find source with iqdb
20150329_183632.jpg
File: 20150329_161914.jpg
Jpg, 9.30 KB, 149×159
edit Find source with google Find source with iqdb
20150329_161914.jpg

Анон, подскажи как решать интегральчики.
Первый пик - те, что у меня не получаются.
Второй пик - не знаю, как компактно решать, кроме нескольких "взятий по частям".
>> No.138797 Reply
>>136767
Нужно быстро без теорем с доказательствами, так сказать, въехать в практическую часть с нуля по
1)Мат логике и теор. алгориитмов
2)теорверу и матстату
Поэтому нужны простенькие учебники (Шеня даже не предлагать, там слишком сложно въехать неподготовленному человеку, да и нет времени въезжать)
>> No.138803 Reply
>>138793
Второй пик – первое это же "табличный" интеграл, надо запомнить формулу. Называется тема "интеграл дробно-рациональной функции". Если вкратце, то соль в том, что отношение двух многочленов раскладывается на сумму мономов, дробей вида N/(ax + b) и дробей вида (Nx + M)/(ax^2 + bx + c). Для этих дробей можно один раз вывести формулу и потом тупо подставлять в неё коэффициенты.
>> No.138804 Reply
>>138393
Бамп. Уже дочитываю библиотечный учебник, который советовал преподаватель, но он немного устарел за 29 лет.
>> No.138807 Reply
>>138803
> раскладывается на сумму мономов
Но там только комплексные корни, разве он раскладывается с действительными коэффициентами?

У кого-нибудь есть идеи, как решать что-нибудь с первого пика?
>> No.138817 Reply
>>138807
Интегралы, интегралы, а я маленький такой
То мне страшно, то мне грустно, то теряю свой покой
1) Разложи дробь, должно получиться: http://www.wolframalpha.com/input/?i=1%2F%28%28x%2B1%29%28x^2%2Bx%2B1%29^2%29
2) Подставляем x = sin(y), раскрываем верхнюю скобку, сокращаем, одно слагаемое тривиально, другое - x/sin^2(x), его по частям (ctg(x)' = -1/sin^2(x))
3) Знаменатель расписываешь, как (ch(x)^2 - sh(x)^2)^2 + 2ch(x)^2sh(x)^2 = 1 + sh(2x)^2/2, дальше замена sh(2x)/sqrt(2) = y и получается производная от арктангенса
4) Подставляешь x = cos(y)^2, тогда (1-cos(y)) / (1+cos(y)) = tg(y/2)^2 (расписать cos(y) по формуле половинного угла через sin(y/2) и сos(y/2))
5) Подставляешь x = ln(sin(y))
>> No.138845 Reply
>>138807
Раскладывай на комплексные, проблем-то. Ещё можно привести к виду dx/(x^2+1), каковой интеграл табличный.
>> No.138854 Reply
>>138807
Ясно, что там только комплексные корни, поэтому первый не раскладывается дальше. Вот как ты там посчитал этот интеграл, запомни, что получилось, и пользуйся дальше. Второй действительно так не раскладывается, как я сказал. Забыл уточнить про кратные множители в знаменателе, если в знаменателе есть, скажем, x^4, то по этому множителю надо раскладывать в сумму A/x + B/x^2 + C/x^3 + D/x^4, то есть тут получается разложение 1/(x^2+x+1) - (x+1)/(x^2+x+1)^2. Дальше считаешь интеграл, как написано тут http://vk.cc/3CYRVK. С другой стороны, можно и через комлпексные, да.
>> No.138879 Reply
Расскажите про методы решения иррациональных неравенств. Единственный, что я знаю - возвести в квадрат и решать как обычные, но тогда с ответом не сходится. В чём отличие?
>> No.138919 Reply
File: 31a.jpg
Jpg, 160.65 KB, 839×1708 - Click the image to expand
edit Find source with google Find source with iqdb
31a.jpg
Что читать по матлогике и теории алгоритмов, после прочтения Шеня?
>> No.138920 Reply
>>138919
http://arxiv.org/list/math.LO/98 - статьи по логике за 1998 год
http://arxiv.org/list/math.LO/15 - статьи по логике за 2015 год
>> No.138921 Reply
>>138919
Роджерс?
>> No.138931 Reply
>>138920
>>138921
Мне скорее по теории алгоритмов, чем по логике.
>> No.138938 Reply
Очень глупый вопрос, надеюсь мне помогут.
Как доказать, что
sqrt(7)-sqrt(2)>sqrt7(7)-sqrt7(2)
sqrt7 - корень седьмой степени
>> No.138940 Reply
>>138938
Перегруппируем члены, чтобы вышло неравенство sqrt(7)-sqrt7(7)>sqrt(2)-sqrt7(2).f(x) = sqrt(x) - sqrt7(x) - возрастающая функция при x>(2/7)^(14/5)(Это можно доказать, взяв производную), а из этого факта неравенство следует по определению.
>> No.138942 Reply
File: mid_77260_7247.jpg
Jpg, 49.45 KB, 500×314 - Click the image to expand
edit Find source with google Find source with iqdb
mid_77260_7247.jpg
>>138940
Спасибо, няша.
>> No.138946 Reply
>>136767
няши, посоветуйте пожалуйста учебник по геометрии для студентов, где описаны векторные пространства, аффинные преобразования, квадрики, и примеры решения заданий на это вот всё более-менее понятным языком, очень сложный в понимании предмет для меня((
>> No.138950 Reply
>>138946
Сам учусь по Кострикину, вроде нормально.
>> No.138956 Reply
>>138931
Роджерс по теории алгоритмов как раз (неудачное название дисциплины).
>> No.138971 Reply
>>138879
Соль в том, чтобы решать равносильное уравнение. Это метод.
В случае иррациональных, то есть где знак корня, уравнение, где оба члена — степени членов данного уравнения, может не быть равносильным данному уравнению.
Мне кажется, ты не шаришь в объектах, в которых решаешь уравнение.
Ты напиши примеры, посмотрим.
>> No.138972 Reply
>>138919
Шень — это ты учебник так назвал? Я не читаю учебники, только изложения от своего лица, то что сами пишут.
>> No.138986 Reply
В общем ребят публикую занятную задачку по теории чисел,я до этого уже публиковал несколько более простых,но в этой задаче идей практически нет,не знаю с какой стороны к ней подходить. И так:у нас есть ряд натуральных чисел,нужно переставить его элементы так,чтобы сумма элементов делилась на их количество,на любой итерации. Например 1,3,5 ...- еденица делится на один,1+3 делится на два, 1+3+5 делится на три...,проблема в том,что нужно ,чтобы рано или поздно ,в переставленном ряду, встретилось любое натуральное число.
В идеале,конечно,нужно хорошее аналитическое решение,но алгоритмы выписывания такого ряда тоже принимаются,желательно с объяснением как он получился.
>> No.138997 Reply
>>138986
Значит, существование взаимооднозначной последовательности, чтобы для любого верхнего предела суммирования сложная сумма при композиции делилась на него?
>> No.139003 Reply
>>138997
Есть мнение,что мощность множества таких рядов,равна мощности множества натуральных чисел.Но,это только мнение.
>> No.139004 Reply
>>138997
Есть мнение,что мощность множества таких рядов,равна мощности множества натуральных чисел.Но,это только мнение.
>> No.139007 Reply
>>136767
Есть такой вопрос, возможно немного глупый, но всё же: в школе был очень злой преподаватель по алгебре, который постоянно кричал на меня за то, что я постоянно ошибался, но я всё равно пытался что-то учить, решать, т.к. хотел стать программистом давняя мечта, почти с пелёнок появилась, а однажды сказал, что я слишком тупой для этого и у меня никогда ничего не выйдет, почему-то тогда меня это сильно задело, я просто забил на предмет, до 11-го класса даже учебников не открывал. В результате я в математике почти абсолютный ноль. Я действительно не способный или ещё можно что-то исправить?
>> No.139008 Reply
>>139007
Ошибся тредом, похоже. Простите.
>> No.139010 Reply
>>138986
>>139003
Скажи-ка, а откуда эта задача взята? У меня есть несколько идей по поводу решения(сразу скажу, что ни одна из этих идей пока к ответу не привела), но все они совсем не элементарные. Как это решить методами более-менее традиционной теории чисел - не представляю даже близко.
>> No.139019 Reply
>>138986
Я не понял. Нужно привести последовательность, в которой встречается любое число, и при любом пределе суммирования этой последовательности сумма делится на предел?
>> No.139025 Reply
>>139019
Насколько я понял, задача такова:

Рассматриваем последовательность x_n такую, что каждый её элемент - натуральное число.
Требуем, чтобы в x_n разным значениям индекса n соответствовали разные натуральные числа.
Требуем, чтобы любое натуральное число входило в x_n с каким-то индексом.
Рассматриваем ряд s_n такой, что s_n = x_1 + x_2 + ... + x_n.
Требуем, чтобы частичная сумма s_n делилась на n для любого значения индекса n.

Доказать, что x_n существует, и привести явное построение.
>> No.139029 Reply
>>139010
Это задачка из какого-то старого сборника по олимпиадным задачам.
Ну,я подозреваю,что задача может решаться не совсем тривиально). Тем не менее интересны будут если не решения,то хотя бы какие-то идеи.
>> No.139034 Reply
>>139029
Школьным олимпиадным задачам?
>> No.139047 Reply
>>139029
Самый тупой алгоритм:
1. X1 = 1;
   пока не надоест:
2. Xn - минимальное число, такое, что частичная сумма Sn кратна n.

При n~1000 такой алгоритм использовал достаточно много чисел до 1000. Нужно только строго доказать, что полученная последовательность содержит все натуральные числа.
>> No.139055 Reply
Посоветуйте задачник по матану (not demidovich). Сейчас читаю Зорича, есть еще книжка Кирилловой-Гвишиани, но там хардкор какой-то.
>> No.139060 Reply
>>139047
Ко мне пришла такая же идея, я выписал членов 50 этой последовательности. Появилась гипотеза: число n появляется не позже, чем на 2n-том месте. Если ты считал тысячу членов, то скажи - на тысяче гипотеза подтверждается?
>> No.139068 Reply
>>139019
>>139025
Я вроде понял. Надо переупорядочить натуральный ряд. То есть нужна биективная последовательность, это я чисто для краткости, конечно, терминология теории множеств тут не необходима.
>>139019
>> No.139070 Reply
>>139055
Бутузов, Шишкин - Математический анализ в вопросах и задачах.

мимо с физфака
>> No.139072 Reply
>>139060
Да.
>> No.139077 Reply
>>139060
Даже больше скажу: число 65533 появляется на 40503-ем месте, так что гипотеза кажется истинной.
>> No.139081 Reply
Если что, это задача появлялась под номером М1517 в Кванте, решение можно найти в одном из журналов за 96-й год
>> No.139085 Reply
>>139081
Не знаю как вы его нашли,но спасибо,считаю тему исчерпанной.
>> No.139088 Reply
Посоветуйте, пожалуйста, учебник по геометрии олимпиадного уровня (только планеметрия).
>> No.139089 Reply
>>139088
Планиметрия*
fixed
>> No.139094 Reply
>>139085
Я нашёл последовательность в OEIS: https://oeis.org/A019444
>> No.139098 Reply
File: 7508t.jpg
Jpg, 29.49 KB, 512×288 - Click the image to expand
edit Find source with google Find source with iqdb
7508t.jpg
У меня возникло желание люто зашарить матан. Да еще и ГОС летом, что тоже стимулирует. Поэтому пока остановлюсь на уровне универской программы. Проблема в том, что я хоть и вроде как знаю матан лучше многих однокурсников, сам прекрасно понимаю, что нихуя не знаю и не помню.
Лекции схоронённые можно достать, но я хочу книги. Возможно, у нас были какие-то книги, но я на учебе вообще не привык пользоваться литературой. За три года обучения я книгу по математическим дисциплинам вообще не открывал.
Хочу книги по всему. От рядов и интегралов до тензорного анализа. От дифуров до функана(из которого я вообще знаю только интеграл Лебега). От урматов до дифференциалов.
Буду очень благодарен за любые варианты.
>> No.139105 Reply
>>139098
Вообще говоря, было бы неплохо, если бы ты слегка сузил свой реквест.
> остановлюсь на уровне универской программы
Какого универа-то? Они, знаешь ли, бывают разные и имеют разные программы.
По упомянутым тобой предметам могу посоветовать "лекции по мат.анализу" Львовского, "математический анализ" Зорича, "лекции об уравнениях математической физики" Шубина, "введение в комплексный анализ" Шабата, книги Арнольда("обыкновенные дифференциальные уравнения", "геометрические методы...", "лекции об уравнениях с частными производными"), "теоремы и задачи функционального анализа" Кириллова-Гвишиани, "лекции по функциональному анализу" Хелемского. Вот насчёт тензоров я тебе ничего не посоветую - всё, что я о них знаю, почерпнуто из "курса дифференциальной геометрии и топологии" Мищенко и Фоменко. Ссылаться на эту книгу - моветон, да и не уверен, что это то, что тебе нужно.
>> No.139107 Reply
File: 1355647125035.jpg
Jpg, 119.09 KB, 937×960 - Click the image to expand
edit Find source with google Find source with iqdb
1355647125035.jpg
Подскажите годную литературу по нечеткой логике и нечетким множествам.
>> No.139108 Reply
>>139105
Физтех. Спасибо за книги.
>> No.139110 Reply
>>139088
Так и называется. "Планиметрия" Прасолов.
>> No.139123 Reply
>>139107
9 выходит на пике? Или я слишком глупый?
Сначала действие в скобках 6/23 выходит, потом делим, 33 == 9?
>> No.139139 Reply
Вопрос к местным. На прошлой неделе я написал контрольную по алгебре, в которой интегральная площадь кривой трапеции вышла отрицательным числом, и я в ответе написал модуль этого числа. Мне засчитали это ошибкой. Почему? Я вычитал интегралы не в том порядке, но ведь если бы я их вычел в другом, вышло бы то же самое число, только положительное, и ответ был бы такой же. Потому что (а-б)=-(б-а).
>> No.139140 Reply
>>139139
Бзик некомпетентного преподавателя, скорее всего. Дай угадаю: оценивала стереотипная женщина, которую уместнее назвать словом "тётка", с трудом отличающая логарифм от интеграла и ничего не слышавшая никогда о теории меры?
>> No.139151 Reply
Господа,посоветуйте жесткую и большущую подборку книг по матану,чтобы можно было ее одним файлом скачать.
>> No.139153 Reply
>>139110
Спасибо
>> No.139154 Reply
File: 3172815.png
Png, 103.77 KB, 600×669 - Click the image to expand
edit Find source with google Find source with iqdb
3172815.png
>> No.139155 Reply
>>139151
Русский:
http://rutracker.org/forum/viewtopic.php?t=630901
Английский:
http://rutracker.org/forum/viewtopic.php?t=2715402
http://rutracker.org/forum/viewtopic.php?t=2829301

Если недостаточно, то существуют библиотеки "колхоз" и "либген".
>> No.139160 Reply
Не могу решить, если решение хотя бы на полстраницы растягивается, то сразу лезет куча ошибок. Все эти диффуры и интегралы просто кошмар, от тренировок прогресса нет.
>> No.139163 Reply
Правда ли, что из обратимости в ассоциативном кольце элемента 1+ab обязательно следует обратимость 1+ba? И почему?
Я что-то ни доказать не могу, ни контрпример не придумывается.
>> No.139166 Reply
>>136767
Так, поясните за формулу анаграммы.
Есть вот слово средство 8 букв, но у нас 2 повторяющиеся буквы. т.е. я беру 8!/2! = 20160 - кол-во анограмм
правильно решил?
>> No.139172 Reply
File: -.png
Png, 18.65 KB, 564×185
edit Find source with google Find source with iqdb
-.png
File: -.png
Png, 30.50 KB, 600×628
edit Find source with google Find source with iqdb
-.png
File: -.png
Png, 48.43 KB, 788×772
edit Find source with google Find source with iqdb
-.png

Анон, прошу у тебя помощи и помочь больше некому. Выручи пожалуйста.
>> No.139176 Reply
>>139172
I. С первой пачкой помогать особо не в чем, считай свертку в лоб. Интегралы x^n * sin x, e^x * sin x - околотабличные, первый - по частям, второй - тоже известным способом решается.

II. Определение отображения знаешь? То, что для каждого эл-та из X есть единственный y = f(x) из Y.

1. Не является.
2. Является.
3. Является.
4. Не является.

Почему так, догадайся из определения, не маленький уже.

III.
Читай определения, ёпрст.
1. ни одно из них
2. только сюръекция
3. только инъекция
4. биекция, т.е. инъекция и сюръекция
>> No.139177 Reply
>>139176
f(0)=0^2=0 - не натуральное жи, братиш, есть жи.
>> No.139178 Reply
Да и в первом листе, видать, не свертка, а композиция, судя по уровню остальных.
>> No.139184 Reply
Ему на сложный ответили >>139172, а мне нет >>139166
   :C
>> No.139190 Reply
>>139184
Твой вопрос неаккуратно сформулирован. К тому же, ты не сказал "пожалуйста".
Но да, решение правильное.
>> No.139193 Reply
>>139163
Кажется, контрипример можно построить так: возьмем ассоциативное кольцо 2х2 матриц над полем из двух элементов. Пусть
S = 0 1 T = 1 1
1 0      0 1
Тогда 1+ST обратимо, а 1+TS - нет. Проверь, я не очень уверен в своих вычислениях.
>> No.139194 Reply
>>139193
>>139193
>>139193
Так, с разметкой случилось что-то невообразимое.
(0 1) = S.
(1 0)

(1 1)
(0 1) = T.
>> No.139212 Reply
>>139176
Спасибо тебе.
>> No.139214 Reply
>>139194
Проверил,
E+ST=(1,1,1,0)
E+TS=(0,1,1,1)
То есть, они не только обе обратимы, но ещё и обратны друг для друга.
>> No.139223 Reply
>>139214
М-да. Ну я же говорил, что не уверен в вычислениях.
Вообще, я тут подумал и понял, что если контрпример и есть, то это точно не матрицы над полем: обратимость матрицы 1+AB равносильна тому, что значение характеристического многочлена матрицы AB в точке -1 ненулевое, а характеристические многочлены АВ и ВА совпадают.
>> No.139226 Reply
>>139223
Я нашел сегодня. Если:
(1+ab)c=1, то
(1+ba)(1-bca)=
1+ba-bca-babca=
1+ba-b(ca+abca)=
1+ba-b[(1+ab)c]a=
1+ba-ba=1.

Но это какой-то уебанский подход, я буквально угадал как обратный для 1+ba должен выглядеть.
>> No.139227 Reply
>>139226
Ты сделал математическое открытие. Поздравляю.
>> No.139230 Reply
>>139227
Не вижу повода иронизировать.
>> No.139231 Reply
>>139230
Это не ирония.
>> No.139232 Reply
>>139231
Пошел ты, дружище.
>> No.139245 Reply
>>139226
Это частный случай формулы enwiki://Woodbury_matrix_identity
Для единичных матриц A, C
>> No.139289 Reply
File: евграфов-1.jpg
Jpg, 72.09 KB, 571×298
edit Find source with google Find source with iqdb
евграфов-1.jpg
File: евграфов-3.jpg
Jpg, 39.55 KB, 572×279
edit Find source with google Find source with iqdb
евграфов-3.jpg
File: евграфов-2.jpg
Jpg, 52.01 KB, 567×323
edit Find source with google Find source with iqdb
евграфов-2.jpg

Анон, помоги, пожалуйста, решить эти задания. Хочу хотя бы несколько намеков на способ решения большинства задач. Не игнорируй и простые уравнения - там только несколько решаются просто с помощью формулы корней из КЧ, для остальных надо угадать гениальный и простой способ.
>> No.139293 Reply
>>139289
Первый пик: ты бы написал, что ли, что такое M(z). Я совершенно не помню, что там в Евграфове так обозначается, мне пришлось лезть скачивать задачник. Первое задание довольно сложное. Распиши формулу, которую предлагается доказывать, через x1, y1, x2, y2(С учётом того, что |z|=x^2+y^2) и возведи её в квадрат, потом распиши квадрат расстояние(попросту как сумму квадратов трёх координат), используя формулы номера 1.38. Сравни эти две записи(развёрнутый квадрат формулу и развёрнутое расстояние) и пойми, как сокращаются члены, которых нет в доказываемой формуле. Второе простое, просто вспомни, что такое М(бесконечность).
Второй пик: уравнения я всё же проигнорирую, уж такую-то мелочь как-нибудь сам. Первая сумма является производной e+e^2+....+e^n. Для такой суммы есть формула, её нужно продифференцировать. Вторая сумма есть (e^2+e^3+...+e^n+e^(n+1))''-(e+e^2+...+e^(n-1)+e^n)', подставляем два раза формулу суммы геометрической прогрессии, дифференцируем.
Третий пик: там есть подсказка, больше сказать, в общем, нечего. Распиши степени е в сумме, которую предлагается подсчитать, в тригонометрической форме, всё и поймёшь. Последний номер пика точно так же решается через арифметическую прогрессию, просто с другим знаменателем.
>> No.139338 Reply
File: sum.jpg
Jpg, 14.44 KB, 467×56 - Click the image to expand
edit Find source with google Find source with iqdb
sum.jpg
>>139293
> Распиши степени е в сумме, которую предлагается подсчитать, в тригонометрической форме, всё и поймёшь.
Спасибо за советы, но здесь я запутался. Как я понял, совет автора воспользоваться именно данной суммой бесполезно. Надо использовать сумму, в которой первый член не 1, а exp(iTheta). По формуле суммы геометрической прогрессии я нашел эту сумму. Получилась громоздкая формула. Но нас интересует только подпоследовательность синусов.

То есть расписали прогрессию как cos t + isin t + cos 2t + isin 2t + ... + cos nt + isin nt. Имеем сумму этой прогрессии в виде дроби. Как из этой дроби выделить сумму синусов?
>> No.139339 Reply
>>139338
> совет автора воспользоваться именно данной суммой бесполезно
> бесполезен
Быстрофикс
>> No.139349 Reply
>>139338
В твоём пике ошибка, в правой части в знаменателе вместо e^(iф)-1 просто e^(iф)
> Как из этой дроби выделить сумму синусов?
Искомая сумма синусов есть мнимая часть суммы геометрической прогрессии, конечно(предлагаю это доказать). Вот подумай, как её выделить. Подсказка: деление есть умножение на обратный элемент, а при умножении мнимая и действительная часть произведения легко выражаются через мнимые и действительные части сомножителей.
>> No.139354 Reply
>>139349
> Искомая сумма синусов есть мнимая часть суммы геометрической прогрессии, конечно(предлагаю это доказать). Вот подумай, как её выделить.
Я это и так знаю. Над остальным сейчас подумаю.
>> No.139357 Reply
>>139293
> Последний номер пика точно так же решается через арифметическую прогрессию, просто с другим знаменателем.
Ты не ошибся? Арифметическая, а не геометрическая?
>> No.139360 Reply
>>139357
Ну геометрическая, конечно. Я вообще там раз сто опечатался.
>> No.139451 Reply
File: 20150414_204247.jpg
Jpg, 97.75 KB, 640×480 - Click the image to expand
edit Find source with google Find source with iqdb
20150414_204247.jpg
доброаноны, помогите пожалуйста привести квадрику к нормальному виду(первое уравнение) и напишите пожалуйста алгоритм приведения квадрики к нормальному виду, ну или просто напишите алгоритм приведения квадрики к нормальному виду))
>> No.139456 Reply
>>139451
А какой вид - нормальный? По-моему итак норм всё, только подобные свести и збс.
>> No.139458 Reply
>>139456
В нормальном вроде xy быть не должно. Для этого поворачивают оси координат и я не помню как это математически выглядит.
мимопроходил
>> No.139459 Reply
>>139451
Приведение квадратичной формы к главным осям?
>> No.139482 Reply
>>139451
Ну как тебе сказать. Сначала выделяешь полные квадраты, то есть убиваешь перекрёстные члены/приводишь матрицу к диагональному виду (x^2 + bxy + cy^2 -> (x+y/2b)^2 + c' y^2. Потом сдвигами x'-> x'+c, y' -> y' + d убиваешь линейные члены, потом немножко масштабируешь и норм.
>> No.139491 Reply
>>139482
>>139458
геометрически эта конструкция означает сначала поворот одной из осей координат, а потом центрирование и растяжение.
медленнофикс
>> No.139514 Reply
File: cat.jpg
Jpg, 73.33 KB, 600×615 - Click the image to expand
edit Find source with google Find source with iqdb
cat.jpg
>>139107
Таки бамп вопросу.
>> No.139530 Reply
File: polar.jpg
Jpg, 17.85 KB, 768×614 - Click the image to expand
edit Find source with google Find source with iqdb
polar.jpg
Анон, нужна помощь. Если даны окружности, расположенные как на картинке или каким-то другим образом (но не с центром в начале координат и не соприкасающиеся с осью Oy), то по какому алгоритму найти пределы изменения полярного угла и полярного радиуса?
>> No.139541 Reply
>>139530
поскольку ответ зависит только от радиуса и расстояния от центра до начала координат, то можно считать, что центр лежит на оси Ox.
>> No.139549 Reply
>>136767
Немного не в тему, но треда для себя найти не смог.
Ни у кого нет текста выводов к таблице по статистике? Буду очень благодарен.
>> No.139550 Reply
>>139541
Нихуя не понял. Приведи пример нахождения пределов изменения полярного угла, когда окружность не содержит в себе начало координат.
>> No.139637 Reply
File: Snap-2015-04-20-at-17.40.49.png
Png, 26.48 KB, 796×288 - Click the image to expand
edit Find source with google Find source with iqdb
Snap-2015-04-20-at-17.40.49.png
доброаноны, помогите, я не понимаю как выводятся формулы на пикче, объясните пожалуйста откуда тут берутся тангенсы
>> No.139638 Reply
>>139637
Делим числитель и знаменатель на (cos x)^2.
>> No.139652 Reply
File: dWc1zTDQyNo.jpg
Jpg, 320.33 KB, 1026×1294 - Click the image to expand
edit Find source with google Find source with iqdb
dWc1zTDQyNo.jpg
>>136767
Учусь на вечернем, дали задание в институте, но оно какое-то странное, в плане того, что граф странный. Как я понял это вес ребер указан или что? Я ничего не понял, почему ребер больше, тогда как их с вершинами соединить.
Как решать сами задания я разберусь. Но вот как сам граф выглядит? Вот например 20
>> No.139653 Reply
File: 312123.PNG
Png, 117.00 KB, 788×126 - Click the image to expand
edit Find source with google Find source with iqdb
312123.PNG
>>139652
Отбой, разобрался, на предыдущей страницы было указано с чего начать.

Буду по-ходу решения тогда спрашивать советы.
>> No.139654 Reply
Эй, почему мои посты удалили
>> No.139671 Reply
Няши, объясните пожалуйста как определять экстремумы функций нескольких переменных, когда переменных 3 и более? Я так понимаю нужно найти как и в случае с 2 переменными частные производные и проравнять их к нулю. Зачем навернон найти вторые производные, а вот как составлять матрицу? Объясните пожалуйста как быть в случае с тремя переменными
>> No.139686 Reply
>>139671
Сначала ты должен найти все особые точки, т.е. точки, в которых все из первых производных равны нулю. Все экстремумы гладкой функции - особые точки, но не все особые точки - экстремумы. Возможны также точки перегиба, сёдла и прочие гадости. Далее для каждой особой точки ты должен найти гессиан(т.е. матрицу вторых производных) функции в этой точке. Если ранг гессиана меньше максимального, то особая точка вырождена, и мы про неё, вообще говоря, ничего не знаем. Если максимален, то остается узнать сигнатуру гессиана. Если сигнатура нулевая - точка является минимумом, максимальная - максимумумом, иначе точка является седловой, т.е. это вообще не экстремум. Согласно лемме Морса, сигнатура гессиана однозначно определяет поведение функции в окрестности невырожденной особой точки. Как-то так.
>> No.139687 Reply
File: 20150422_213905.jpg
Jpg, 104.25 KB, 640×480 - Click the image to expand
edit Find source with google Find source with iqdb
20150422_213905.jpg
>>139686
эмм, не совсем понимаю, давай на примере, вот у меня есть такая функция трёх переменных, найдены её частные производные первого и второго порядка, найдена подозрительная на экстремум точка, вот как дальше составлять матрицу?
>> No.139688 Reply
>>139687
> вот как дальше составлять матрицу
Если ты имеешь в виду гессиан, то на (i,j)-том месте будет стоять смешанная производная (d^2 f)/(dxi*dxj). В твоём случае гессиан попросту диагонален, все его элементы положительные, так что сигнатура равна нулю, т.е. это минимум.
>> No.139689 Reply
>>139688
я не понимаю((, напиши пожалуйста саму матрицу
>> No.139691 Reply
>>139687
Ну и вообще в твоём случае вся эта наука не нужна. Твоя функция есть просто сумма квадратов(плюс-минус константа), у неё единственный глобальный минимум(который находится элементарно как только ты приведёшь функцию к виду суммы квадратов) и нет максимумов и сёдел.
>> No.139692 Reply
>>139689
Чёрт, ты безнадежен.

2 0 0
0 2 0
0 0 2

Что тут неясного? Ты же уже выписал все производные.
>> No.139693 Reply
File: 20150422_222301.jpg
Jpg, 108.50 KB, 640×480 - Click the image to expand
edit Find source with google Find source with iqdb
20150422_222301.jpg
>>139688
   то есть в случае если функция является функцией 3 и больше переменных, то нужно составлять матрицу Гессе? И она также как и в случае функции 2 переменных должна быть больше нуля?
И как определить, является это точкой минимума или максимума? ТОже смотреть по элементу матрицы с индексами 11?
>> No.139694 Reply
>>139692
спасибо большое)), так а как узнать является точка точкой минимума или максимума? если det>0 то это точка минимума, а если <0, то точка максимума?
>> No.139696 Reply
>>139693
> И как определить, является это точкой минимума или максимума
Я уже говорил - нужно смотреть на сигнатуру гессиана. Точка не обязательно является минимумом или максимом, есть множество промежуточных вариантов - сёдел. В твоём случае гессиан диагонален, все элементы на диагонали положительны, поэтому точка - минимум. Если бы сигнатура была равна трём(в общем случае - числу переменных), то это был бы максимум. Все промежуточные варианты, где сигнатура принимает значения от единицы до (число переменных)-1 являются сёдлами, т.е. вообще не экстремумами. Определитель же гессиана о типе особой точки нам, вообще говоря, ничего не говорит.
>> No.139698 Reply
>>139686
объясни как найти сигнатуру гессиана? и вообще что это такое?
>> No.139700 Reply
>>139698
О чёрт, я только сейчас сообразил, что немного напутал с формулировками. Извини. Не сигнатура, а индекс. Так вот, индексом квадратичной формы -(x1)^2 - (x2)^2 -... - (xk)^2 + (xk+1^2 +...+(x_n)^2 называется число k. Любую квадратичную форму можно единственным образом привести к каноническому виду, поэтому индекс определен однозначно. Гессиан(думаю, определение гессиана вопросов не вызывает) - симметричная матрица, поэтому с ним связана единственная квадратичная форма. Индекс гессиана - индекс этой квадратичной формы.
Например:
-1 0
   0 1 соответствует форме y^2 - x^2 и имеет индекс 1.

1 0 0
0 -1 0
0 0 -1 имеет индекс 2.
>> No.139701 Reply
>>139700
я не понимаю, как найти этот индекс, это определитель матрицы? объясни пожалуйста на примере моей матрицы, как ты узнал, что индекс моей матрицы равен нулю?
>> No.139702 Reply
>>139701
Это НЕ определитель матрицы. Как я уже сказал, это количество -1 на диагонали после приведения матрицы квадратичной формы к каноническому виду. Если такое определение тебе непонятно, то, думаю, у тебя большие пробелы в линейной алгебре.
Индекс твоей матрицы равен нулю потому что её канонический вид - единичная матрица 3х3, на диагонали которой нет минус единиц.
>> No.139706 Reply
Ясно, спасибо большое, няш
>> No.139735 Reply
Анон, как понять пучки? Ни хуя не получается, я уже раза три к ним подступался и всё тщетно, какой-то пиздос, никогда таких проблем не было.
>> No.139737 Reply
Анон, мне нужно понять линейную алгебру, часть физики и матана.
Но прорешивать задачи я не хочу.
Есть ли какие нибудь структурированные учебники, где по каждой теме дается, например, по 100 полностью решенных задач?
>> No.139745 Reply
>>139735
Какую книгу(книги) юзаешь?
Какой у тебя математический бекграунд?
По моему опыту, нужно изучить определения, а потом постараться найти как можно больше примеров, связывающих пучки и другие математические понятия(определение гладкого многообразия через пучки, пучки как пучки сечений векторных расслоений, структурная теорема для квазикогерентного пучка на афинной схеме - навскидку). Тогда наступит некоторое понимание.
>> No.139767 Reply
File: 14288428336410.png
Png, 191.11 KB, 803×1001 - Click the image to expand
edit Find source with google Find source with iqdb
14288428336410.png
Это университетская программа или выше?
Без окончания универа можно и не лезть даже в это?
>> No.139768 Reply
File: 4706554.jpg
Jpg, 55.69 KB, 450×600 - Click the image to expand
edit Find source with google Find source with iqdb
4706554.jpg
>> No.139772 Reply
File: tumblr_nn3f3tK1IY1r922azo1_500.gif
Gif, 2031.09 KB, 500×278 - Click the image to expand
edit Find source with google Find source with iqdb
tumblr_nn3f3tK1IY1r922azo1_500.gif
Если у меня диффур(ли?) типо вот такого:
Adx+B(1+Cx-Dy)dy=0
x,y - независимые обе.
Чо делать? Вольфрам думает что игрек зависит от икс и пишет хуйню, а как ему объяснить я не знаю.
>> No.139773 Reply
>>139772
Маленько напутал, там интеграл вообще взять надо. Но я чому-то обосрался с того, что от икса по иреку надо интегрировать. Или мож там просто за константу считать и все?
>> No.139775 Reply
>>139773
> Маленько напутал, там интеграл вообще взять надо
> Adx+B(1+Cx-Dy)dy=0
Гм. Я пока не очень понимаю твоей записи. Было б лучше, если бы ты сфотографировал задание.
Могу предположить, что это дифференциальная форма, которую нужно интегрировать по кривой, а потом из условия равенства интеграла нулю найти коэффициенты.
>> No.139795 Reply
>>136767
Добротян, хочу научиться алгебре. Открыл Кострикина-Манина, увидел слово абелева группа, закрыл. С чего надо начинать?
>> No.139796 Reply
>>139795
Кострикин-Манин - это книжка для продвинутых. Есть трёхтомник, который Кострикин написал в одно лицо. Называется "Введение в алгебру". Начни с него.
>> No.139803 Reply
>>139795
Зачем закрыл-то? Алгебра, тащемта, и начинается с групп.
>> No.139804 Reply
>>139795
Есть такая книжка - Михалёв, "начала алгебры". Это плохой учебник, вообще говоря, но для начинающих, которые продвинутые учебники оценить не могут, он подходит довольно хорошо. difficulty curve весьма деликатная в нём.
>> No.139809 Reply
>>139795
> тян
> математика
>> No.139819 Reply
>>139803
Потому что начиналось не с того, что такое абелева группа, а давалось определение, которое её использовало.
>>139796
Спасибо.
>>139804
Я могу в сложные учебники, но не могу начинать учить с середины.

>>139809
> chan
Доброматематики всем ITT.
>> No.139842 Reply
>>139819
> > chan
Читается: чан. От слова чанелл, канал. Тян это тян, пиздоноска.
>> No.139845 Reply
>>139842
Знаю. А chan в ромадзи читается как тян.
Мне нравится воспринимать Доброчан как антропоморфную сущность- Добротян. Когда-то были рисунки Добротян, но она была не очень популярна, а потом художник-куны ушли или вымерли.
>> No.139846 Reply
>>139845
> Мне нравится воспринимать Доброчан как антропоморфную сущность- Добротян
Конечно, ведь ирл вокруг тебя никаких антропоморфных сущностей нету, не то что добрых, не то что тян.
>> No.139871 Reply
File: Снимок-экрана-от-2015-04-29-15:30:40.png
Png, 24.30 KB, 579×420 - Click the image to expand
edit Find source with google Find source with iqdb
Снимок-экрана-от-2015-04-29-15:30:40.png
Выручай анон. Не могу решать такое.
>> No.139872 Reply
>>139871
Ты не пробовал прочитать определение и сопоставить то что видишь с определением?
>> No.139873 Reply
>>139872
Я в этом вообще не могу понять ничего.
>> No.139874 Reply
>>139873
Напиши определение и укажи, какие именно слова ты в нем не понимаешь.
>> No.139875 Reply
>>139874
А можно сказать, что всё?
Анон, мне просто ответы бы знать. Если можешь помочь, то выручи пожалуйста.
>> No.139876 Reply
>>139875
Здесь кафедра математики, а не клуб помощи тем, кто не умеет читать. И, признайся, ты ведь даже не пыталась, а?
>> No.139877 Reply
>>139876
Почти весь курс решен. Потом его пришлось оставить, а сейчас вернутся к нему и закрыть его. Это последний на курсе тест из трех заданий. Остальные задания решены. Ты ведь не бросишь меня, правда?
>> No.139878 Reply
>>139877
Что за курс? Как ты попала на этот курс? Как ты, черт возьми, решила все задания, кроме того, которое проверяет твое знание базовых определений, без которых ты не можешь вообще работать с булевыми функциями?
>> No.139879 Reply
>>139878
Первый курс грубо говоря информатики, на который удалось поступить. Там были не только булевые функции, там вообще каша из разных тем.
>> No.139880 Reply
>>139879
Ну, раз уж тебе удалось туда поступить, то и это ruwiki://Конъюнктивная_нормальная_форма ты прочитать можешь, не так ли? (обрати внимание на примеры. Еще раз обрати, и задумайся, почему ты не могла этого сделать сама).
Но вообще, мне правда стало интересно, что это за информатика и где ей так отвратительно учат.
>> No.139881 Reply
>>139880
И на том спасибо... T_T
>> No.139882 Reply
>>139881
Эх ты... Выходит, ты и правда этих определений в глаза не видела. А уже анону глазки строишь, чтобы он за тебя все сделал. Терпеть не могу таких людей. Но это же доброчан, так что исправляйся, неняша. И приходи, когда поймешь что сейчас ты живешь неправильно.
>> No.139883 Reply
>>139882
> А уже анону глазки строишь, чтобы он за тебя все сделал.
И ничего не строю. Просто прошу по человечески помочь и надеюсь это не составит труда.
>> No.139888 Reply
Да какого хуя вы анонов тян называете? ДВМ тред в /b.
>> No.139899 Reply
Анон, есть одна задача: вычертить область 1 <= |z - i| < 2, Re z <= 0, Im z > 0 на комплексной площади. Я сделал чертеж на бумаге, но мне надо сдавать индивидуалку в печатном виде. В какой программе можно делать такие чертежи, чтобы можно было чертить окружности и оси пунктиром и подписывать оси Re z и Im z?
>> No.139907 Reply
>>139899
> на комплексной плоскости
>> No.139912 Reply
>>139899
> чертёж на бумаге
> комплексная плоскость
I lol'd

Почти в любой программе, в которой можно чертить графики. От Exel до Маткада.
>> No.139918 Reply
>>139912
"Почти в любой программе, в которой можно чертить графики" -> "В любой программе, где можно чертить"
даже в sprint layout'е можно
>> No.139932 Reply
Господа, будьте добры, объясните гуманитарию физический или какой еще сможете придумать смысл транспонирования матриц, и, особенно, зачем в макроэкономических моделях встречается произведение квадратной матрицы на саму себя транспорированную
>> No.139933 Reply
>>139932
Извини, но твой вопрос слишком обширен. Частным случаем матриц являются обычные вещественные числа как матрицы 1x1. Мне придётся рассказать тебе про все аспекты использования чисел, и это будет только частью ответа на твой вопрос.
>> No.139934 Reply
>>139933
Будет ли ответ полон, если скажу, что нужна эта операция для поиска обратных матриц, а также для решения задач по оптимизации? Или нечто еще?
>> No.139936 Reply
>>139934
Матрица, на главной диагонали которой стоят 1, а все остальные элементы 0, называется единичной и обозначается I.
Например,
1 0 0
0 1 0
0 0 1
Матрица A' называется обратной матрице A, если AA' = I.
Обратные матрицы могут быть только у квадратных матриц.

Пусть есть квадратная матрица A. Типичный алгоритм получения обратной ей матрицы таков.
1. Посчитать определитель матрицы A, det A. Если det A = 0, то обратной матрицы нет.
2. Составить матрицу алгебраических дополнений, так называемую "союзную матрицу".
3. Транспонировать союзную матрицу и умножить каждый её элемент на число 1/detA.
Полученная матрица будет обратной для A.

Но саму матрицу в этом алгоритме транспонировать не требуется же. Странно.
>> No.139938 Reply
>>139932
Транспонированная матрица есть матрица сопряженного оператора в базисе, двойственном к данному. Линейные функционалы на векторном пространстве V составляют линейное пространство V* одинаковой с V размерности. Значение функционала y на векторе x обычно обозначается за <x;y>. Если у нас задан базис x1,...,xn в V, то можно построить двойственный базис в V, функционалы которого y_1,...,y_n удовлетворяют условиям <x_i;y_i>=1,<x_j;y_i>=0 при i=/=j. Так устанавливается изоморфизм между V и V, вот только таких изоморфизмов много, и в разных координатах они записываются по разному(в отличие от этого V и V** изоморфны канонически). Так вот, если A - линейный оператор из V в V, то условие <Ax;y>=<x;By> для любого вектора x и функционала у однозначно определяет оператор B на V*. Оператор B в базисе y1,...,yn имеет матрицу, получающуюся при транспонировании матрицы оператора A в базисе x1,...,xn.

Это, пожалуй, самый общий ответ. В частных задачах транспонирование может применятся и для совсем других целей.
>> No.139939 Reply
>>139938
Запорол разметку, всё время забываю, как работает курсив. Ну ладно, думаю, читаемо.
>> No.139940 Reply
>>139938
> устанавливается изоморфизм между V и V
Между V и V*, конечно.
>> No.139941 Reply
>>139932
Самый простой прикладной пример.
Есть у нас система линейных уравнений. A*x=B
Очевидно(хех), его решением будет х = B * A^(-1)

Не обязательно линейных(просто я первокурсник), кстати. Но вообще, в физике и экономике часто встречаются системы уравнений, а их часто хорошо решать через матрицы.
>> No.139942 Reply
>>139941
Эй, с каких это пор матрицы коммутируют?
Решение это х = A^(-1) * B.
>> No.139943 Reply
>>139941
Собственно в этом и заключался вопрос. Только ради этого или нет.
Курс высшей математики я прослушал, но примеры не приводились из прикладных наук, а матрицы для прикладных целей только сейас понадобились.
В одной книге "Основания экономического анализа" Самуэльсона, которой, к сожалению, на руках у меня нет, использовались транспонированные матрицы. Если они использовались лишь в этих целях, то вопрос снят и буду ими в дальнейшем пользоваться
>> No.139946 Reply
>>139938
>>139936
Немного уточню вопрос.
Сложение может выступать в значении "Или", умножение в значении " И", но транспонирование только для поиска обратных матриц и решения задач оптимизации, основанных на них? Я так должен себе это представлять?
>> No.139950 Reply
>>139946
Да нет, конечно. Часто это вообще вопрос удобства. Хочешь писать вектор столбцом - применяй оператор слева. Хочешь писать его строкой - транспонируй матрицу оператора и применяй справа. А для нахождения обратных матриц оно вообще не нужно, ибо метод Крамера (тот, который тебе описали) слишком медленный, и все нормальные люди и программы используют метод Гаусса.
Вообще, чего ты к этому транспонированию так привязался? Поменять строки и столбцы местами не велика наука.
Не представляю, впрочем, что там у вас за логика на матрицах в этой вашей экономике.
>> No.139951 Reply
>>139950
Думал, что какой-то сакральный смысл в этом есть. Извиняюсь за глупый вопрос.
В Матлабе, Octave или Python с Numpy есть ведь функции работы с матрицами? Через что субъективно удобнее всего работать?
>> No.139953 Reply
>>139951
Лично я использую питон, но я пишу свои библиотеки для алгебры по мере ее изучения, так что про numpy много сказать не могу, кроме того, что он показался мне не слишком удобным. Многие нахваливают Wolfram Mathematica, но она, как и матлаб, денег стоит. Попробуй разные вещи и реши, что лично тебе удобнее.
>> No.139955 Reply
>>139946
> Сложение может выступать в значении "Или", умножение в значении " И"
Не понимаю тебя. Если рассматривать матрицы над полем из двух элементов, то такое толкование уместно, но навряд ли ты об этом. Боюсь, ты путаешь с какими-то полузабытыми вещами из информатики.
>> No.139956 Reply
>>139955
Признаю, иногда так для меня удобнее, а привёл их исключительно как пример интерпретации, ни в коем случае, на строгость не рассчитывая.
>> No.139967 Reply
Может кто объяснить смысл второй по важности константы в математике? Число е, для чего оно нужно? Правильно ли я понимаю, что это число используется для перевода десятичного логарифма в натуральный или как-то наоборот? И еще, у меня возникают вопросы с натуральным логарифмом, скажем, у нас есть lg1, ясно, что 10^0=1, а как быть с натуральным логарифмом, если туда надо будет подставлять какие-нибудь цифры? ln1=0, e^0=1, а, если будет ln2, как это считать? И вообще, как лучше всего разбираться с наибольшим и наименьшим значением производной log и ln функций? Просто считать, а потом подставлять значения? Или может есть какой-нибудь хак. Извиняюсь за глупые вопросы...
>> No.139969 Reply
>>139967
> смысл
Ознакомься со всеми теоремами, в которых фигурирует e. Это и будет смыслом.
Начни с ruwiki://Формула_Эйлера
> как это считать
Разложением в ряд. Например, в ruwiki://Ряд_Тейлора
С помощью ряда значение функции в конкретной точке можно посчитать до любого заданного наперёд знака после запятой.
ln(2) = ln(1+1) ≈ 
≈ 1 - 1/2 + 1/3 - 1/4 + 1/5 - 1/6 ≈ 
≈ 0.6
>> No.139981 Reply
>>139967
Качество и количество твоих вопросов наводит на мысль, что ты толком и не разбирался ни в чём, а поспешил сразу спросить в интернете.
> Правильно ли я понимаю, что это число используется для перевода десятичного логарифма в натуральный или как-то наоборот?
Ну и для этого тоже, хотя говорить, что в этом состоит "смысл" числа е - бессмыслица. У чисел нет смысла, как не бывает "смысла стула" или "смысла камня". Камни существуют, могут быть факты о камнях, могут быть некие принципы и теории, связывающие камни с другими камнями или иными предметами и явлениями, но вообще говорить о "смысле камня" глупо. Вот скажи, какого рода ответ ты ожидаешь на вопрос о "смысле" некоторого числа?

Ты хотя бы контекст задай, процитируй конкретное место из учебника или лекции, которое тебе непонятно, тогда можно будет что-то сказать.
> если будет ln2, как это считать?
А как посчитать log e? Как посчитать log 5, если уж на то пошло? Десятичный логарифм кажется тебе "более понятным", хотя с ним есть точно такие же вопросы. На вопрос про конкретно ln 2 тебе уже ответили, впрочем.
> И вообще, как лучше всего разбираться с наибольшим и наименьшим значением производной log и ln функций?
Этот вопрос крайне плохо сформулирован. Если речь идёт о (ln x)' и (log x)', то про них можно узнать в любом учебнике анализа, это даже в большинстве школ проходят, и оценить их не составляет никакого труда(т.е. на прямой, они, конечно не ограничены, но на любом подмножестве прямой, не содержащем некоторую окрестность нуля - да). Если речь идёт о том, что называют "логарифмической производной", то тут нет и не может быть никаких общих методов, и я настоятельно рекомендую тебе всё-таки сказать, логарифмическую производную которой функции ты собираешься оценивать.
>> No.139985 Reply
>>139967 d(e^x)/dx = e^x
>> No.139986 Reply
>>139981
> log x
кстати, это логарифм по какому основанию?
>> No.139988 Reply
>>139986
log - 10
ln - e
lg - 2 (иногда)
>> No.139990 Reply
>> No.139997 Reply
>>139988
log без основания - то же самое, что ln.
ln - e
lg - 10
lb - 2
>> No.139998 Reply
>>139997
> log без основания - то же самое, что ln.
log без основания пишут там, где основание и так очевидно читающему. В статьях, связанных с Computer Science это скорее всего будет 2 Обозначения lb я вообще ни разу не видел, в случае с дискретным логарифмированием это образующий элемент группы, и так далее.
>> No.140000 Reply
>>139998
Вольфрамальфа по дефолту заменяет ln на log.
http://www.wolframalpha.com/input/?i=ln+7
>> No.140001 Reply
>>140000
Потому что именно это чаще всего имеют в виду ее пользователи. Никакого отношения к записи в других местах это не имеет.
>> No.140010 Reply
Здравствуйте. Не подскажете как найти объём цилиндра, согнутого в кольцо, если я знаю площадь поперечного сечения и длину прямой, проходящей сквозь центр цилиндра на протяжении всего кольца ?
>> No.140011 Reply
>>140010
Это ты так тор что-ли обозвал?
>> No.140012 Reply
>>140010
Цилиндр, который согнули в кольцо, деформирован и больше не является цилиндром. Нужная тебе вещь называется полноторие. Объём полнотория вычисляется по формуле V = 2(π^2)R(r^2).

Площадь поперечного сечения обозначим s. Понятно, что s = πr^2.
Длину известной прямой обозначим l. Понятно, что l = 2πR.
Отсюда V = π(r^2)2πR = sl.
Это было ожидаемо.
>> No.140013 Reply
>>140011
Тор - поверхность полнотория.
>> No.140030 Reply
http://www.math24.ru/%D0%B4%D0%B2%D0%BE%D0%B9%D0%BD%D1%8B%D0%B5-%D0%B8[...].html

Как в примерах 4 и 5 сектор S получается?
>> No.140041 Reply
Анон, в книге Виро я прочитал, что топологию может освоить даже школьник, и чтобы читатель не влюбился в этот размел математики, потому что в математике есть еще много интересного. А в чем, собственно, красота топологии? Какие-то множества оче много теории множеств, определения. Для чего это? Чем глубже изучаешь, тем интереснее?
>> No.140042 Reply
>>140041
Ты ещё не научился видеть за языком объекты, которые описаны этим языком. Научишься - поймёшь.
>> No.140044 Reply
>>140042
В анализе я вижу за языком объекты, а в топологии не могу увидеть. Как этому научиться?
>> No.140046 Reply
>>140044
В теоретико-множественной, или общей топологии(которую ты, скорее всего, и учил, раз жалуешься на засилье теории множеств) действительно не так уж много красивого. Красота появляется на более продвинутом уровне: в алгебраической топологии, анализе на многообразиях, дифференциальной и алгебраической геометрии. А общая топология это просто основа для более сложных наук, самостоятельной ценности она не имеет.
>> No.140057 Reply
Бамп. Стоит ли учить все определения в математике наизусть? Я всегда их просто разбирал до полного понимания, и в таком интуитивно понятном виде они находились у меня в памяти. А не так давно понял, что этого мало, что я не могу сходу дать точное определение и вспомнить какое-то определение в точности.
>> No.140059 Reply
>>140046
> анализе на многообразиях
Довольно странно учить общую топологию и не задеть хоть немного анализ на многообразиях.
>> No.140060 Reply
>>140059
Думаю, ты немного путаешься в терминологии. Под общей топологией обычно понимается материал примерно от определения топологического пространства до конструкции фундаментальной группы. В большинстве курсов общей топологии(учебник Вербицкого, тот же Виро, Бурбаки, Келли) многообразия даже не определяются.
>> No.140062 Reply
>>140060
Скорее, я смотрю с позиции физика, которому анализ более интересен, чем абстракции над абстракциями. Но с другой стороны, анализ — весьма естественная область применения подобных вещей, не?
>> No.140063 Reply
>>140062
> подобных вещей
Подобных чему?
> Скорее, я смотрю с позиции физика
С какой бы позиции ты не смотрел, многообразия в курс общей топологии в подавляющем большинстве случаев не входят.
>> No.140064 Reply
>>140063
> не смотрел
ни смотрел
fix.
>> No.140077 Reply
>>140063
У кого не входят?
Я не математик. Однако были топологические многообразия (только двумерные. В больших размерностях только про гроссманиан сказали). Гладкие и римановы тоже были, но не в курсе общей топологии. Без этих многообразий приличной механикой не получится заниматься, например.
>> No.140078 Reply
>>140077
Офигеть. Это круто.
>> No.140079 Reply
>>140077
> У кого не входят?
У математиков. Как дела обстоят у физиков, я не знаю, но я всё же исходил из того, что мы обсуждаем математические книги и математические вузы. Тащемта, я спорил с изначальным утверждением о том, что странно учить общую топологию, не задев при этом анализ на многообразиях. Для математика в этом ничего странного нет, поэтому это утверждение меня задело и я не мог его не оспорить.
>> No.140099 Reply
>>136767
Здравствуй, кафедра математики, пишу свой первый курсач второй курс, сразу встал перед проблемой актуальности задачи: реализации алгоритма деления многочленов в поле k[X1..Xn]. Вопрос, собственно вот в чем: а зачем мне их делить? Понятное дело, что делить их приходится, но в голову не лезет ничего конкретного, кроме базисов Грёбнера, для которых достаточно деления одночленов.
Консультация только в четверг, а хотелось бы на выходных начать делать. Можно было бы пока что пропустить введение, но я решил сначала спросить у мудрого анона.
>> No.140100 Reply
>>140099
Это важно для работы в конечных полях и кольцах многочленов. Они строится как остатки от деления на неприводимый многочлен, обратные элементы в них находятся по алгоритму Евклида, который тоже завязан на алгоритм деления. И тем же алгоритмом Евклида можно например определить, имеют ли два многочлена общие корни, или имеет ли многочлен кратные корни.
Вообще, смотри в сторону криптографии. Там многочлены используются много где, и скорость работы с ними зело важна.
>> No.140101 Reply
>>140100
Спасибо.
> смотри в сторону криптографии
Ну да, это вроде как профиль мой. Но дело в том, что на деле мы пока толком ничего не проходили именно с практической точки зрения, потому сейчас сижу и ищу.
>> No.140107 Reply
>>140100
> Они строится как остатки от деления на неприводимый многочлен, обратные элементы в них находятся по алгоритму Евклида, который тоже завязан на алгоритм деления.
Просто для справки: случай многих переменных сильно отличается от случая одной переменной. Ничего не стоит привести кучу примеров, где профакторизовав по неприводимому многочлену ты поля не получишь. Более того, если переменных больше одной, а поле коэффициентов алгебраически замкнуто(например, это поле комплексных чисел), то из теоремы Гильберта о нулях прямо следует, что фактор по одному многочлену полем быть не может. А всё потому, что для многочленов от нескольких переменных нет такой вещи, как тождество Безу, и расширенный алгоритм Евклида не работает.
Простой пример: в кольце C[x,y,z,t] многочлены f=xy и g=zt совершенно очевидно взаимно просты. Но вот найти два ненулевых многочлена u и v, чтобы выполнялось равенство uf + gv = 1 у тебя не получится, думаю, понятно почему.
>> No.140116 Reply
>>136767
Посоветуйте годный учебник по формальной логике. Скачал Чёрча, вроде неплохо. Однако область относительно новая, всего то сто лет, может чего нового придумали.
>> No.140128 Reply
>>140099
> Вопрос, собственно вот в чем: а зачем мне их делить?
Если до тебя ко второму (!) курсу не дошел практический смысл математических понятий и алгоритмов, лучше выпились. Небось поступал, чтобы бабло грести, а приходится заниматься "фу че за хуйня эта математика нахуй она в жизне нужна)))"?
>> No.140130 Reply
>>140101
> Ну да, это вроде как профиль мой
> вроде как
Ты даже не знаешь, зачем на специальность поступил и для чего тебе приходится изучать эту математику. Ваную твое разочарование и бугурт, когда математика "не пригодицо в жызне)))" и ты просто потратишь 5 лет впустую. Потому что математика - для тех, кто любит отвлеченную игру ума и испытывает потребность в интеллектуальных удовольствиях. То есть не для тебя.
>> No.140132 Reply
Анон, я ленивый человек. Я не хочу пробираться сквозь доказательства и решать задачки. Я хочу лишь набраться знаний по верхушкам. Какие ты знаешь книги, где всё на пальцах, разжуют и рот положат всякие любопытные факты. Вроде: глядите, а тут у нас очень интересный изоморфизм. Можете себе представлять (ко)касательное расслоение, как то-то. Пучки и схемы — это просто!
>> No.140133 Reply
>>140132
Нет пути. Математика - это упорный труд. Зачем тебе набираться по верхам, если это никогда тебе не доставит такого удовольствия, как понимание всех глубинных механизмов теорий?
>> No.140134 Reply
>>140132
А зачем тебе всё это, попросту говоря?
Что тебе дадут эти знания по верхам, особенно если речь идет о такой сложной и абстрактной области, как теория схем?
Вообще, у меня когнитивный диссонанс от твоего поста. Ты знаешь такие слова как "кокасательное расслоение", "схема", "пучок". Наверное, ты знаешь также и соответствующие определения и некоторые логические связи между ними. Такие знания случайно обрести невозможно(ну или я не понимаю как). Значит, ты всё же занимался математикой какое-то(значительное, раз уж добрался до схем)время. Неужели с занятиями не пришло понимание, что лёгкого пути в математике нет? И как ты дошел до нынешнего уровня, если ленив и не любишь решать задачи?
>> No.140135 Reply
>>140134
На самом деле я покривил душой, это не для меня. Цель одна: заинтересовать и мотивировать адептов юных. Всем в любом возрасте было интересно читать предисловия умных книг, так почему бы не составить одну сплошь из таких предисловий с интересными фактами и самой мякоткой следствий. Вот я и интересуюсь, может есть такая?
>> No.140136 Reply
>>140135
Уровня "схемы и пучки - это просто!" я никогда не видел. Думаю, такого вообще не бывает. Если говорить о алгебраической геометрии, то ближайшее к тому, что ты описал(из известного мне) - лекции Манина(недавно частично переизданные под названием "введение в теорию схем и квантовые группы") и обзоры Данилова("алгебраические многообразия и схемы", "когомологии алгебраических многообразий"). Алсо, обзор Шафаревича про алгебру. Вообще, если тебя интересует максимальная плотность красивых и важных результатов и некоторая неформальность, то обзор - именно тот жанр, на который стоит обратить внимание.
Но все вышеперечисленные тексты всё равно непросты.
>> No.140137 Reply
>>136767
Может быть, не в тему, но, пожалуйста, поясните за мат аппараты, которые используются в криптографии. А то техническую часть я уже облазил вдоль и поперёк, хочется освоить математическую часть, от которой сейчас бросает в первобытный ужас.

Спасибо.
>> No.140139 Reply
>>140137
Конкретизируй. Какие именно вопросы тебя интересуют?
>> No.140140 Reply
File: 4dde1b4755882c935dae94d0712b0fc3[1].png
Png, 22.49 KB, 536×154 - Click the image to expand
edit Find source with google Find source with iqdb
4dde1b4755882c935dae94d0712b0fc3[1].png
>>140139
Ну например, открываю я пособие по криптографии и вижу там рилейтед. Вот что это, как это? Описания этого нет, значит, я уже должен это знать, но что это?
>> No.140141 Reply
>>140140
Хз, спрашивай у автора пособия. Я сходу могу придумать аж три интерпретации, и во всех фигня получается.
>> No.140142 Reply
>>140140
Без контекста абсолютно непонятно что это. Какие-то авторские обозначения.
Хорошие статьи ко криптографии можешь найти здесь: https://www.iacr.org/cryptodb/archive/
Ищешь по авторам, видишь знакомую фамилию (автор интересующей тебя криптосистемы, например), читаешь. При необходимости следуешь по ссылкам, которые используются в статье.
Если тебя интересует что-то конкретное, я может быть смогу подкинуть материал.
>> No.140143 Reply
>>140142
На руссише бы.
>> No.140144 Reply
>>140140
Что за книга?
>> No.140145 Reply
>>140144
Бабаш А. Криптография
http://rghost.ru/6FpRfzMlC
>> No.140146 Reply
>>140142
Там написано про какие-то конечные поля. Мне ещё в треде криптографии рассказали, что эти конечные поля лежат
в основе криптографии, но проблема в том, что они тоже как-то не берутся, если ещё дальше истоки?
>> No.140147 Reply
>>140146
Ты знаешь определение поля и определение конечности? Просто чтобы понимать твой уровень.
>> No.140148 Reply
>>140147
На данный момент, нет.
>> No.140149 Reply
>>140145
Судя по оглавлению, книга не сильно хороша, только 6я глава выглядит интересно, но она потребует хорошего знания алгебры. Попробуй Шнайера "прикладная криптография" для начала.
>>140146
Лидл-Нидеррайтер - это все что тебе нужно, чтобы познать конечные поля (на разумеется, при условии, что ты хотя бы на базовом уровне понимаешь всякий линал). Но поля используются в первую очередь для ассиметричной криптографии, симметричная может обойтись и без них, хотя операции в поле бывают хороши для повышения криптостойкости системы при комбинировании с другими операциями (см. алгоритм IDEA например).
Потом еще будут эллиптические кривые, но это отдельная история.
И вообще, учи английский. Хорошего материала на русском зачастую просто нет.
>> No.140150 Reply
File: капча-поработать.png
Png, 0.81 KB, 300×20 - Click the image to expand
edit Find source with google Find source with iqdb
капча-поработать.png
>>140148
Ну заебись просто. Т.е. даже слово "конечный" тебе непонятно?
Извини, но это очень плохо. Википедию хоть пробовал читать?
ruwiki://Конечное_поле
Там, в принципе, все определения даны. На данный момент ты вообще ничего не знаешь, капча намекает на то, что тебе нужно сделать.
>> No.140151 Reply
>>140149
Попробую разобраться. Спасибо большое за информацию, буду пробовать её осваивать.
> > Лидл-Нидеррайтер
Эта книга даже имеется у меня.
>> No.140152 Reply
>>140151
> Эта книга даже имеется у меня.
Вот и читай. Или мне отдай, а то бумажную сейчас хер достанешь.
>> No.140153 Reply
>>140152
Вот и буду читать.
>> No.140155 Reply
>>140136
А как искать обзоры? У Шафаревича, например, в названии книги нет слова "обзор".
>> No.140156 Reply
>>140155
А я не знаю, лол. Я назвал три обзора, которые имели отношение к моему полю деятельности, и которые я изучал, а дальше...Тут разбираться надо. Ну кстати, отличный обзор Рохлина по топологии обзором и назывался.
>> No.140219 Reply
File: f4.gif
Gif, 0.80 KB, 145×45 - Click the image to expand
edit Find source with google Find source with iqdb
f4.gif
Слушайте, какое практическое назначение имеет формула среднего значения распределения информации?
>> No.140225 Reply
>>140219
В силу одной из теорем Шеннона через неё(т.е. энтропию распределения, формула которой изображена на твоём пике) оценивается(причём и сверху, и снизу) стоимость оптимального кода для данного распределения вероятностей. Это самое простое. Дальше следуют всякие теоремы о блочных кодах и универсальных блочных кодах, в которых также через неё выражаются всяческие оценки.
>> No.140228 Reply
>>140219
Если по-простому: ты не можешь сжать данные сильнее, чем позволяет их энтропия.
>> No.140233 Reply
File: 377px-Sovershenno-noviy.jpg
Jpg, 36.89 KB, 377×599 - Click the image to expand
edit Find source with google Find source with iqdb
377px-Sovershenno-noviy.jpg
>>140228
Смогу!
>> No.140252 Reply
>>140228
А я правильно понимаю, что оптимальный способ сжатия существует, но алгоритмически ненаходим (по аналогии с Колмогоровской сложностью)?
мимо
>> No.140267 Reply
>>140252
Нет, неправильно. Любой современный алгоритм сжатия асимптотически близок к оптимальному.
>> No.140268 Reply
>>140252
В случае кодирования - находим. Называется кодом Хаффмана. Вообще, хоть теоремы Шенонна и дают ограничения на оптимальность, но эта оптимальность весьма специфическая - измеряется только средняя длина слова в коде. Точно так же энтропия - тоже средняя величина. Аналогии с колмогоровской сложностью проводить не стоит, хотя бы потому, что колмогоровская сложность сколько-нибудь общей последовательности зависит от её длины, а не от каких-то усреднённых параметров.
>> No.140320 Reply
File: big_10.jpg
Jpg, 40.75 KB, 562×375 - Click the image to expand
edit Find source with google Find source with iqdb
big_10.jpg
Товарищи, а можно по-быстрому, как понять, имеет ли СЛУ решения над кольцом? Есть там какие аналоги определителей и проч.?
>> No.140327 Reply
>>140320
Над которым кольцом? Разные кольца принципиально по-разному устроены. Общего ответа быть не может, конечно.
>> No.140338 Reply
>>140327
Z_n, n составное, разве такие кольца обладают какими-то специфичнскими свойствами, кроме означенных в 6 аксиомах?
>> No.140339 Reply
>>140338
Над коммутативным кольцом матрица обратима тогда и только тогда, когда её определитель обратим. Например, над целыми числами обратимы только матрицы с определителем +1 либо -1. В твоем случае определитель должен быть взаимно прост с n.
>> No.140340 Reply
>>140339
Где можно об этом прочитать?
>> No.140341 Reply
>>140339
Да и про отстутсвие свойств я совершенную глупость сказал, извини.
>> No.140342 Reply
>>140340
Берешь любой учебник линейной алгебры, смотришь на формулы элементов обратной матрицы(там, где они выписываются через алгебраические дополнения и определитель), внезапно понимаешь, что эти формулы работают в любом коммутативном кольце, если определитель обратим.
>> No.140344 Reply
>>140342
Да, вроде работает, но все равно, чтобы убеждаться таким образом, надо иметь абсолютно наидичайшее чутье, потому что от приятного очень просто в таком случае убедиться.
>> No.140351 Reply
Доброчан, посоветуй книгу с методами решения олимпиадных задач. Именно с методами, а не сборник олимпиадных задач. Буду очень благодарен.
(Да, и если доброчанька посоветует какую другую полезную олимпиадную литературу по математике - буду очень рад. Сам пока что занимаюсь по "Пять колец"(районный уровень, дальше ниасиливаю) и Горбачёву.
>> No.140352 Reply
>>140351
Канель-Белов, Ковальджи "как решают нестандартные задачи".
Тут, впрочем, есть некоторые оговорки:
1)большинство методов решения олимпиадных задач представляют из себя огрубленные и упрощенные методы высшей математики. Многие олимпиадники при решении олимпиадных задач пользуются матаном, линейной алгеброй и прочей математикой университетского уровня, а потом переводят решение на "школьный" язык. Это я к тому, что знание методов, изложенных в книжках для школьников, полной картины не даст.
2)умение решать олимпиадные(как и любые другие)задачи приходит с практикой. Знание методов само по себе дает мало. Вот если потренироваться лет эдак пять - тогда точно взлетит.
>> No.140355 Reply
>>140352
> Пять лет
Увы, мне через два года поступать.
Но делать нечего, хоть и поздно, но начинать всё равно надо.
>> No.140356 Reply
>>140355
Не лучше ли два года поготовиться к экзаменам и университетской пргограмме? По-моему, в такой ситуации выбор очевиден.
>> No.140358 Reply
>>140356
А по-моему нет. Я хочу поступить в хороший вузфизтех. Физика на физтех-олимпиаде невероятно лёгкая, а математику оттуда решать я не умею.
Если получить диплом - 100 баллов сразу. В том, что я получу диплом по физике я не сомневаюсь, а за эти два года математику подучить, возможно, выйдет.
И ещё - если ты говоришь, что олимпиадные методы решения задач - упрощённые методы из высшей математики, то разве решение олимпиад не поможет в дальнейшем и не будет подготовкой к университетской программе?
>> No.140359 Reply
Анон, где можно найти выведение формулы параметрического задания отрезка? Мне впадлу запоминать то, что я не понимаю, хотелось бы разобраться, почему формула именно такая.
>> No.140360 Reply
>>140358
Ну, кстати, в данном конкретном случае это разумно, в физтех берут именно по олимпиадам.
> не будет подготовкой к университетской программе?
Лучшей подготовкой к университетской программе будет изучение университетской программы. При этом ты также получишь +20 к решению олимпиадных задач, т.к. олимпиадные задачи из высшей математики и составляются. Но это моё личное мнение, многие, наверное, не согласятся.
>> No.140361 Reply
>>140360
> олимпиадные задачи из высшей математики и составляются
Ну на школьных олимпиадах есть геометрия, которая вообще отдельная от математики наука.
Не факт, что полезно тратить время на то, чтобы учиться её решать.
>> No.140362 Reply
>>140359
Отрезка на действительной прямой или в произвольном векторном пространстве?
В любом случае, ни разу не видел, чтобы эти формулы выводились. Как правило, их считают очевидными.
>> No.140366 Reply
>>140361
Ну я же оговаривался про "большинство" задач. Олимпиадная геометрия не только бесполезна, но ещё и бесперспективна для самостоятельного обучения - кмк, без преподавателя там ловить нечего. Так что о ней даже и говорить нечего. С другой стороны, в олимипиаде физтеха такой дряни вроде нет.
>> No.140372 Reply
File: матрица.jpg
Jpg, 92.20 KB, 1102×690 - Click the image to expand
edit Find source with google Find source with iqdb
матрица.jpg
Анон, как в ебаном ворде набрать пикрелейтед? Я уже думаю набирать латексом и принтскринить, потому что средств говна для математики не хватает.
>> No.140376 Reply
>>140372
Почему бы не писать сразу в латексе?
Если честно, я не в курсах - неужели где-то от математиков требуют набирать курсовые в ворде?
>> No.140378 Reply
>>140376
> Почему бы не писать сразу в латексе?
Потому что сразу никто не сказал, а потом уже, когда курсовая была наполовину набрана, мне в калтакте сказали про латекс.
>> No.140379 Reply
>>140378
Алсо, посоветуйте лучший билд этого редактора TeX, Latex
>> No.140385 Reply
File: Untitled0.jpg
Jpg, 298.34 KB, 1024×768
edit Find source with google Find source with iqdb
Untitled0.jpg
File: Untitled1.jpg
Jpg, 179.21 KB, 1024×768
edit Find source with google Find source with iqdb
Untitled1.jpg

>>140372 молча
>> No.140455 Reply
Доброчан,прошу,помоги.
a^2(b+c-a)=b^2(a+c-b)=c^2(a+b-c). a,b,c - ненулевые. Доказать,что a=b=c
>> No.140470 Reply
>>140455
Уфф, насилу решил. Откуда задача-то?
Перепишем первое равенство как b^3 - a^3 = ab^2 + cb^2 - a^2b - a^2c.
Разложим правую и левую части на множители:
(b-a)(a^2 + ab + b^2) = c(b-a)(b+a) + ab(b-a);
(b-a)(a^2 + b^2 - bc - ac)=0;
Выпишем аналогичные равенства всех пар переменных:
(b-a)(a^2 + b^2 - bc - ac)=0;
(b-c)(b^2 + c^2 - ab - ac)=0;
(a-c)(a^2 + c^2 - ab - bc)=0;
Предположим, что ни одно из равенств (a-b) = 0, (b-c) = 0, (a-c) = 0 неверно. Тогда верны все три равенства (a^2 + b^2 - bc - ac)=0, (b^2 + c^2 - ab - ac)=0, (a^2 + c^2 - ab - bc)=0.
Многочлены a^2 + b^2 - bc - ac, b^2 + c^2 - ab - ac и a^2 + c^2 - ab - bc - однородные. Значит, без ограничения общности можно считать, что a=1(т.к. а заведомо ненулевое).
Получаем:
1 + b^2 - bc - c=0;
b^2 + c^2 - b - c=0;
1 + c^2 - b - bc=0;
Вычитаем третье уравнение из первого, получаем (b-c)(b+c+1)=0. Если нулевая первая скобка, то получаем противоречие с предположением, сделанным ранее. Если вторая, то подставляя c = -1 -b во второе уравнение, получаем квадратный трёхчлен, не имеющий корней. Значит, верно либо (a-b)=0, либо (b-c)=0, либо (a-c) = 0. Т.к. всё симметрично, возьмём первое равенство. Подставляя a=b во вторые и третьи уравнения, получаем все три требуемых равенства.
Подозреваю, возможно более простое и элегантное решение из соображений однородности и симметрии(все три выражения переходят друг в друга при перестановке переменных), но что-то оно не пришло.
>> No.140475 Reply
>>140470
Вступительная в один лагерь
Простите меня,это единственная задача нерешённая
>> No.140501 Reply
>>140470
>>140455
Ну, к слову. Если здесь ещё кто-то есть, может порешаем задачки? А то тред какой-то очень по делу, люди получают ответы на свои вопросы и уходят, а общения нет.
Могу предложить парочку понравившихся мне.
>> No.140506 Reply
>>140501
Дело в том, что и я могу предложить, и каждый тут может предложить, но интересы редко пересекаются, так что...
>> No.140507 Reply
>>140506
Так предложи, а я посмотрю. Я многое готов решать.
>> No.140509 Reply
>>140508
> вне зависимости от выбранной системы координат.
Это предложение лишнее же.
С ходу решать эту задачу не возьмусь, т.к. формулировка кажется мне противоречивой: в первом предложении говорится о векторном пространстве над полем, но далее мы говорим о k-арных функциях шестизначной логики. Над каким полем мы рассматриваем (Z/6Z)^k как векторное пространство? Если не рассматривать экзотические варианты, то это скорее модуль над кольцом, а тогда я с трудом представляю себе определение "общего положения".

Моя задача: пусть G - группа, f:G -> G - такое, что f(a)f(b)f(c) = f(x)f(y)f(z) для любых a, b, c, x, y, z таких, что abc=xyz=1. Доказать, что существует h из G такое, что hf - гомоморфизм, т.е., что hf(xy) = hf(x)h*f(y) для всех x, y.
>> No.140512 Reply
File: -14321604916780.jpg
Jpg, 55.64 KB, 1270×228 - Click the image to expand
edit Find source with google Find source with iqdb
-14321604916780.jpg
Как это решить?
>> No.140560 Reply
>>136767
Подскажите литературу, где описываются с примерами разные методы построения полиномов Жегалкина (методом неопределенных коэффициентов и при помощи СДНФ), методы Квайна и карты Карно, ну и все такое прочее. Меня интересует техника решения подобных задач.
>> No.140579 Reply
>>140560
Википедия.
>> No.140580 Reply
>>140512
1)попросить автора прокомментировать, что за хуйню он написал.
>> No.140584 Reply
File: aaa.gif
Gif, 0.40 KB, 46×33 - Click the image to expand
edit Find source with google Find source with iqdb
aaa.gif
Как можно разложить эту дробь на простейшие?
И еще, как, блядь, преодолеть эту ненависть арифметических выражений? Все эти трюки-хуюки а теперь решим интеграл ололо
>> No.140585 Reply
>>140584
Определи простейшие дроби как дроби вида x/(1-cx) и скажи, что задача решена.
>> No.140589 Reply
>>140585
У меня прав нет.
>> No.140590 Reply
>>140589
Мы, Царь Император, сим тебе их дали.
>> No.140591 Reply
>>140590
У вас их тоже нет.
>> No.140592 Reply
>>140591
Нет есть.
>> No.140593 Reply
>>140592
А я уже понял как разложить.
>> No.140594 Reply
>>140579
Мне впадлу разбираться в теории на вики, мне бы просто задрочить технику. Терпеть не могу мат. логику, мне больше пользы дало чтение материалов для школьников типа что такое необходимое и достаточное условие. Тем более что у нас мат. логика - это вовсе не логика, а параша из дискретки с вычислением всяких СДНФ и полиномов Жегалкина, в то время как на самой дискретке идет теория графов.
>> No.140597 Reply
Доброаноны, подкиньте чего-нибудь по началам проективной геометрии, пожалуйста.
>> No.140599 Reply
File: погн.png
Png, 57.65 KB, 660×632 - Click the image to expand
edit Find source with google Find source with iqdb
погн.png
Как такое получается?
>> No.140602 Reply
>>140599
Это попросту уравнение градиента длины траектории(на самом деле дифференциала, но от дифференциала градиент отличается только скалярным умножением на ковектор, и о градиенте рассуждать легче). Длина траектории растёт при движении по окружности против часовой стрелки, поэтому градиент направлен по касательной к окружности(которая и записана в скобке). По модулю градиент равен производной длины как функции единственного аргумента theta, отсюда первый множитель. d theta - ковектор, при умножении на который из градиента получается дифференциал.
>> No.140604 Reply
>>139795
Начать сначала можно, читая одну из книг.
Варден ван дер, Алгебра
Куликов, Алгебра и теория чисел
Ленг, Алгебра (сразу идёт шире других)
Александров П. и др., Энциклопедия элементарной математики. Часть I. Арифметика (упрощённо, читать статьи про множество и последующие про числовые системы)
Можешь обратиться к Феферман, Числовые системы, довольно интересно. Причём можно начать с приложения, где приведены аксиомы неформальной теории множеств. По-моему, в своё время это наиболее уместное чтение, чем всё остальное, названное мной.
>>140057
Если исходить из того, что определения — это сокращения, то запоминать неестественно. Просто знаешь, что имеешь в виду, и в ходе исследования применяешь определение или "даёшь определение", что для меня звучит синонимично. А вот "сходу дать точное определение" звучит нелепо, только если это не значит "отчеканить перед экзаменатором". Но это уже не имеет отношения к исследованию. Видимо, такие ситуации возникают из-за навязывания всяких "теоретических минимумов" и всякого такого, но по мне так либо сейчас погружён в тему и можешь обсудить, либо нет. И если когда-то был погружён, то это не значит, что сейчас отчеканишь.
>>140116
Читал Карри, бросил по середине, и уже давно нет аппетита. Только в последнее время мелькала мысль, но сейчас не читаю. Смотри в предыдущем треде по слову "Карри", я там писал про неё.
И ещё, подумай о книгах не как об учебниках, а как о трактатах.
>> No.140612 Reply
>>140604
> Ван дер Варден
> Куликов
> Ленг
Не слушай этого говноеда.

Завтра ищешь в интернете книжку Шафаревича "Основные понятия алгебры". Похуй если ничего не поймешь. Затем ищешь Городенцев "Алгебра для студентов-математиков" и изучаешь от корки до корки. Потом зубришь, именно, сука, вызубриваешь учебники Кокс, Литл, Ши "Идеалы, многобразия" и Прасолов "Геометрия", чтобы от зубов отскакивало. Когда докажешь свою первую теорему, по пути изучив кодирование информации на алгебраических кривых, скачиваешь и изучаешь любой пейпер зарубежных математиков в области алгебраической геометрии - рикамендую Stiechtenoth. Как опубликуешь свой труд о кодах, лежащих на границе Грассмана, можешь идти дальше - тебя ждет увлекательный мир R-модулей и когомологий. Отсос потреблядских выблядков / просто неудачников типа твоих однокурсников, которые поступили на чисмат, чтобы просто отличаться от гуманитариев))), которые сосут хуй по жизни не заставит себя ждать и уже через пол года ты будешь получать такие суммы, что любая баба будет течь при одном упоминании твоей зарплаты.
>> No.140745 Reply
Как можно найти область сходимости ряда Лорана, в котором z^n в делителе?
>> No.140751 Reply
>>140745
О сходимости ряда Лорана говорят тогда, когда сходятся и его главная, и его правильная часть. Главная часть сходится вне некоторого круга(возможно, всюду, кроме некоторой точки), правильная - в некотором круге(возможно, в одной точке). Пересечение двух этих областей даёт кольцо сходимости(возможно, пустое).
>> No.140752 Reply
File: 4.jpg
Jpg, 220.18 KB, 1280×960 - Click the image to expand
edit Find source with google Find source with iqdb
4.jpg
>>140612
У этой пасточки есть генератор?
>> No.140778 Reply
File: 666.png
Png, 3.98 KB, 108×89 - Click the image to expand
edit Find source with google Find source with iqdb
666.png
>>140751
Мне ясно как найти область сходимости если ряд в своем нормальном виде Cn * (z - a)^n, но не ясен пикрилейтед. Сумма от 0 до бесконечности, но рассматривать ее как правильную часть нельзя из за 1 / z^n. За главную тоже считать не получится, так как сумма должна быть от -1 до бесконечности, и даже если переверну дробь обратно чтобы z^n был в числителе, получится от 0 до -бесконечности.
>> No.140781 Reply
>>140778
> За главную тоже считать не получится, так как сумма должна быть от -1 до бесконечности
Ты правда не понимаешь как обойти эту проблему?
>> No.140783 Reply
>>140778
> За главную тоже считать не получится, так как сумма должна быть от -1 до -бесконечности
fix
>>140781
Перевернуть дробь, получить сумму от 0 до -бесконечности ( (-n)! * z^(-n) ) / 2^n.
Но сумма получится от 0 а не от -1 же. Можно вычесть значение когда n = 0, но тогда это будет другой ряд, нет?
>> No.140784 Reply
>>140783
Я опять неправильно написал выражение, но ты понял.
>> No.140785 Reply
>>140783
> Можно вычесть значение когда n = 0, но тогда это будет другой ряд, нет?
Скажи, это первое упражнение на сходимость рядов в твоей жизни?
Да, ряд станет другим. Пусть у нас сначала есть ряд А. Мы вычли из него член нулевой степени, получился ряд Б. А теперь подумай, как сходимость ряда А связана со сходимостью ряда Б. Подсказка: это упражнение на пределы уровня первой половины первого семестра матана.
>> No.140787 Reply
>>140783
> Перевернуть дробь, получить сумму от 0 до -бесконечности ( (-n)! * z^(-n) ) / 2^n.
Уж не знаю, что ты имеешь в виду, но куда естественнее сделать замену переменной w = 2/z. Получится ряд с членами w^n/n!, т.е экспонента. Фактически, это и есть ответ.
>> No.140802 Reply
Научите решать рандомно уравнения 3 степени.

Вот, намприер

х^3-20х^2+131х-232=0
>> No.140806 Reply
>>140802
enwiki://Cubic_function#Roots_of_a_cubic_function
Решить можно, но формулы уродливые
>> No.140836 Reply
Для испытания шерстяной ткани на прочность произведены две выборки объемом в 10 и 12 образцов. Средняя прочность оказалась равной 135 и 136 г при исправленных выборочных дисперсиях 4 и 6. Считая выборки извлеченными из нормальных совокупностей, определить при уровне значимости 0,01 существенность расхождения между средними в обеих выборках.
анон, намекни, как решать это. посидел пару часов с лекцией на руках и не понял как. ну или годной литературы подкинь.
>> No.140860 Reply
Математик-кун,а что ты думаешь об НМУ?Стоит ли вообще там учиться?Реально ли учиться параллельно с другим вузом?Сложно ли сдать первую сессию?Можешь посоветовать материалов про подготовку к ней?
>> No.140862 Reply
>>140860
Хорошее место. Стоит, безусловно. Да, реально. Да, сложно. Могу.
Материалы:
http://www.mccme.ru/free-books/yaschenko/v08book-08.pdf
http://www.mccme.ru/free-books/yaschenko/v08book-09.pdf
>> No.140863 Reply
>>140862
За сколько времени это все можно прорешать?
>> No.140865 Reply
>>140863
Сначала прочитай это. Вечер на одну книгу, вечер на другую.
>> No.140866 Reply
>>140865
>>140865
Спасибо.Буду готовиться.
Кстати,куда можно поступать чтобы реально было добираться и учиться?Говорят мехматовцы там учатся.А из Долгопрудного реально добираться?
>> No.140868 Reply
File: 14260212640680.jpg
Jpg, 114.99 KB, 795×875 - Click the image to expand
edit Find source with google Find source with iqdb
14260212640680.jpg
>>140866
Ну, Городенцев в прошлом году говорил, что на Алгебру-2 половина студентов ходит с матфака ВШЭ. Я и сам в этом году поступлю на МФ, тоже буду в НМУ ходить. С МФТИ ты вряд ли сможешь совмещать, у них на первом курсе в понедельник и четверг пары с утра до позднего вечера, грузят сильно домашками, лабами.

Материалы выше выглядят очень годно. Лично я летом прорешаю «Теорему Абеля» Алексеева, «Наглядную топологию» Прасолова прочитаю, а дальше буду штудировать Зорича + АнтиДемидовича + Натанзона. Ещё валяются на полке разные книжечки.
Алгебра: Городенцев, Винберг, Шафаревич, Генфальд, Кострикин, Кострикин-Манин
Топология: Виро, Васильев, цифровой Вербит на ноуте
Геометрия: Прасолов-Тихомиров, Акопян-Заславский «Геом. свойства КВП»
Анализ: Львовский, комплан Шабата зачем-то
Этого должно хватить с избытком на первые 2-3 семестра.

Математик-кун, поясни за семинары в НМУ. Как они выглядят? Я решаю листок, а в НМУ у меня его кто-то индивидуально принимает, или для всех показывают разборы на доске? Расскажи про процесс, в общем.
>> No.140869 Reply
>>140868
> Генфальд
Лол, втф. Гельфанд, конечно же.
>> No.140870 Reply
>> No.140871 Reply
>>140870
Вот, я почему-то про Гендальфа думал, хех...
>> No.140873 Reply
>>140865
> Вечер на одну книгу, вечер на другую
> 248 страниц
>> No.140874 Reply
>>140873
Анон, возможно, имел ввиду, что стоит для начала просто познакомиться с содержанием, прочитать определения и формулировки теорем, а уже потом их запоминать и прорешивать упражнения.
>> No.140875 Reply
>>140862
Каков бугурт у меня. Походу, они в 8 классе проходили то, что мы проходили на первом курсе моего картофанно-водочного матфака.
>> No.140876 Reply
>>140875
В какой-то из годов, в 57-й школе несколько 11-классников успели листками добраться до многообразий.
>> No.140878 Reply
>>140876
Каких многообразий? Если алгебраических, то там все просто, если не рассматривать всякие факторизации.
>> No.140879 Reply
>>140876
А где бы достать все эти листки в правильном порядке?
>> No.140880 Reply
>>140878
>>140879
http://www.mccme.ru/free-books/57/davidovich.pdf
Ну, вот. В последнем доп. листке — многомерный анализ.
>> No.140881 Reply
>>140880
О, я оказывается всю обязательную часть довольно неплохо знаю. Не зря пять лет в вузике просидел, лол. А вот дополнительная за 10-11 класс будет очень полезна, спасибо.
>> No.140888 Reply
>>140875
Ну потому что программа первого курса большинства матфаков вполне усваивается даже не очень продвинутыми восьмиклассниками, по крайней мере большая её часть.
>> No.140892 Reply
Сел доказывать задачу @ После получаса размышлений покрасивее нарисовал букву в условии
Тру стори.
>> No.140898 Reply
Анон, как разобраться в логике необходимых, достаточных и равносильных условий? У меня с этим серьезные проблемы. В школе в нашем обычном классе заставляли просто вычислять под водовку и картофанчик, а в матклассе заставляли вычислять в два раза больше. Математическую культуру никто не прививал.
>> No.140900 Reply
>>140898
Пусть из A следует B.
Тогда B называется необходимым условием (синоним: признаком) для A, A называется достаточным условием для B.
Утверждение, что A эквивалентно B, значит, что из A следует B и из B следует A.
То есть утверждение, что A необходимо и достаточно для B, значит, что A эквивалентно B.
Необходимое и достаточное условие называется критерием.

Пусть из A следует B. Если B неверно, то неверно и A. Если B верно, то A может быть как верным, так и неверным.
Таким образом, если необходимый признак X не выполняется, то не выполняется и X. Если же необходимый признак X выполняется, то X всё же не обязано выполняться.

Пусть из A следует B. Если A верно, то верно и B. Если же A неверно, то B всё равно может оказаться верным.
Таким образом, если достаточный признак X выполняется, то и X тоже выполняется. Если же достаточный признак X не выполняется, то X всё же может выполняться.
>> No.140901 Reply
>>140900
Я все это и нагуглить могу. От чтения подобных паст ясности в голове не прибавляется. Мб пытаться искать эти форму суждений в теоремах?
>> No.140903 Reply
File: image012.jpg
Jpg, 22.72 KB, 378×328 - Click the image to expand
edit Find source with google Find source with iqdb
image012.jpg
>>140901
Я не могу думать за тебя.
>> No.140904 Reply
>>140901
Я не понимаю, чего ты не понимаешь. Все кристально прозрачно же.
Приведи пример, который вызывает у тебя затруднения.
>> No.140905 Reply
File: bioshock-infinite-elizabeth-wallpaper-14.jpg
Jpg, 258.05 KB, 2560×1440 - Click the image to expand
edit Find source with google Find source with iqdb
bioshock-infinite-elizabeth-wallpaper-14.jpg
>>140868
> Математик-кун, поясни за семинары в НМУ. Как они выглядят? Я решаю листок, а в НМУ у меня его кто-то индивидуально принимает, или для всех показывают разборы на доске? Расскажи про процесс, в общем.
Поясните, пожалуйста. А то я хикка, стесняюсь ходить на семинары... Мне нужно знать, как там всё будет...
>> No.140907 Reply
>>140903
Но ты можешь подсказать мне, как улучшить свое мышление.
>> No.140908 Reply
>>140907
Я не тот анон, но попробуй порешать задачки логические. Из ЕГЭ по информатике порешай системки логических уравнений. Инструкций к ним в интернетах тонны.
>> No.140909 Reply
>>140905
Зачем тебе НМУ?
>> No.140911 Reply
>>140909
В хорошей аспирантуре смотрят на пройденные магистерские курсы. Видел как-то парня, который прошел в Принстон. Так у него было под три десятка оных + статьи в рецензируемых журналах американских и пара каких-то премий. Имо, такого не получится добиться без доп. образования, а в аспирантуре хочется всё-таки учиться топовой. Да и просто круто, что в НМУ задачки сложные, курсы разнообразные очень. На МФ спецкурсы все для старшекуров все и довольно базовые:
http://math.hse.ru/anonsy2015
Ну и вдруг тусовочка студентов понравится, найду друзей там.
>> No.140930 Reply
Анон, посоветуй учебник алгебры с задачами, по которому можно было бы активно заниматься. Учебник Городенцева сложноват - для решения задач оттуда недостаточно информации из самого учебника.
>> No.140935 Reply
>>140898
Вот, держи книжку: http://www.alleng.ru/d/math/math117.htm
B — необходимое условие для A означает, что A ⇒ B (или что если неверно B, то неверно и A)
Достаточное условие — наоборот, B ⇒ A (если верно B, то неизбежно верно и A)
Равносильность означает, что либо A и B оба верны, либо оба неверны.
>> No.140936 Reply
Уважаемые математики, подскажите, где можно достать аглицкий перевод "General trattato di numeri et misure" Nikkolo Tartaglia
>> No.140938 Reply
>>140904 рискну предложить чур не ржать громко такой пример: в какой зависимости находятся исход шахматной партии и цвета фигур?
>> No.140942 Reply
>>140911
> что в НМУ задачки сложные
Видел я эти задачки. Они такие же, как и в вузе. Но ты сам решай, нужна ли тебе эта ебля.
>> No.140943 Reply
>>140938
> в какой зависимости
> исход шахматной партии и цвета фигур
Ни в какой.
>> No.140945 Reply
>>140943
Ну, если учесть, что белые всегда ходят первыми... Где-то видел статистику, что белые таки выигрывают чаще.
>> No.140946 Reply
>>140945
> зависимости
Зависимости никакой.
>> No.140951 Reply
>>140945 один попался, лол
>>140946 да, цвет - это свойство материала фигур, он не является необходимым условием. так вот, а как узнать, что является необходимым и достаточным для определения шахмат?
>> No.140956 Reply
>>140951
> цвет - это свойство материала фигур
В случае шахмат важно то, что белые ходят первыми. Какое ещё свойства материала, болезный?
>> No.140960 Reply
Анон, что делать, если у меня такая проблема: разберу конец доказательство - забываю начало; вернусь к началу - забываю конец? Как эта хуйня лечится?
>> No.140961 Reply
>>140960
Рисованием на отдельной бумажке упрощённой схемы доказательства.
>> No.140963 Reply
>>140956 свойство материала, из которого состоят реальные фигуры, поглощать ЭМИ. ты хочешь сказать, что если я покрашу фигуры в зелёный и красный, это уже не будет игрой в шахматы, так что ли?
>> No.140964 Reply
>>140963
Тебе лет 12? Нужно же понимать, что «белые» — абстрактное понятие, обозначающее фигуры, которые ходят первыми. Хоть из языков мертвых девственниц фигурки вылепи и в форме президентов США, а белые всё равно будут ходить первыми, и конь будет ходить буквой Г, и король будет двигаться на 1 клеточку в каждую сторону. Даже если вместо клеточек будут овалы, лишь бы доска была топологически эквивалентна квадрату на плоскости.
>>140961
Или прокачав рабочую память упражнениями. Но твой способ охуительно гениален, я всё время голову насиловал, лол.
>> No.140965 Reply
>>140964
Топологически - избыточно сильное требование. Достаточно изоморфизма нотаций.
>> No.140966 Reply
>>140964 то есть ты утверждаешь, что если холодильник белый, он ходит первым, да? а куда он ходит? а как, у него же ног нету?
>> No.140967 Reply
>>140966
В шахматах нет холодильника. Перестань.
>> No.140968 Reply
>>140967 тут выше утверждалось, что "белые" - абстрактное понятие
может стоит прекратить эти беспомощные детские попытки родить очередность ходов и наборы фигур?
>> No.140970 Reply
>>140968
Хватит, это не смешно. Очевидно, что это — часть правил игры. Абстрактные, да. Можешь играть и холодильниками, но тогда «белыми» они будут не в физическом смысле, а в смысле правил.
>> No.140971 Reply
>>140968
Белые здесь употребляется в значении "те, кто ходят первыми". Шах и мат, тролль.
>> No.140989 Reply
>>140971
> Шах и мат, тролль.
Осторожней, у него от такой фразы может и мозг заклинить!
>> No.140996 Reply
За лето хочу прорешать «Матан в 57-й шк.» от корки до корки. Если кто-то хочет — присоединяйтесь, будем проверять друг друга, подсказывать в сложных местах.
>>140880
>> No.140997 Reply
>>140996
Присоединяюсь, няша. Пиши на anon.rerait@gmail.com
>> No.140998 Reply
>>140997
Да ну нахрен. Дали вам имиджборду, общайтесь, спрашивайте, так нет, всю активность надо перенести в закрытые конфочки. Чтобы только что пришедший анон не знал что его ждет.
Я вот тоже присоединиться хотел. Только мне сначала сессию закрыть надо. И что, смогу я через две-три недели найти, где вы там общаетесь? Нет. Смогу ли я узнать, что вы вообще что-то делаете? Нет. Видимо, я как обычно останусь один.
>> No.140999 Reply
>>140998
Это судьба таких как мы, бро. Быть в одиночестве. Добра тебе.
>> No.141001 Reply
>>140998
Ну что же ты, я не хотел никого обидеть. Просто скооперироваться с заинтересованными людьми по почте куда проще, чем постоянно рефрешить тред. Я тоже не могу заняться прямо сейчас, потому что сдаю экзамены, так что, если тебе тоже хочется решать, то пиши на эту же почту :3
>> No.141003 Reply
>>141001
Потом зайдет в тред еще кто-то, увидит, что месяц назад кто-то собирался заниматься годными вещами. Как он узнает, что вы действительно ими занимаетесь? Напишет тебе на почту? Плохой способ. Да, кооперация - это хорошо. Но кооперация должна быть открытой. Чтобы вновь пришедший легко мог а)узнать о ее существовании б)узнать о текущем положении вещей в)влиться в нее.
Я не буду участвовать в ваших закрытых конфочках. Там я все равно не могу сказать ничего такого, что я не мог бы сказать здесь.
И да, я понимаю, что Доброчан может оказаться не самой удобной платформой для обсуждения. Если вам нужна более удобная платформа, откройте ее для чтения всем, и дайте ссылку на нее здесь.
>> No.141004 Reply
>>141001
Да ну, почтой же неудобно. Лучше уж конфу вк или в скупе. Их и найти проще. А можно и прямо в треде, чтоб в любом момент желающие могли присоединиться. Да и случайные аноны, которые прям всё не захотят прорешивать, смогут подсказать с отдельными задачами.
>> No.141006 Reply
File: yDqh3Qkt65A-(1).jpg
Jpg, 55.32 KB, 581×279 - Click the image to expand
edit Find source with google Find source with iqdb
yDqh3Qkt65A-(1).jpg
Возник вопрос по матлогике, изучаю самостоятельно как могу по гуглу и разным книжкам.
Допустим задание 1 - все понятно, строю таблицу истинности и смотрю что да как, деля на куски.
Во втором задании я немного не понял, что значит упростить используя равносильности и как ДОКАЗЫВАТЬ при помощи таблиц?
В третьем как я понял правую или левую часть по каким нибудь законам аля де-моргана преобразуем пока не станут похожими?
В четвертом задании все тот же вопрос, что значит равносильно?
В пятом я тупо подставляю значения и смотрю результат?
>> No.141010 Reply
>>141006
> как ДОКАЗЫВАТЬ при помощи таблиц?
Строишь две таблицы истинности, если они одинаковые - ты доказал. Это и называется равносильностью.
Во втором - берешь и упрощаешь. Так например (~y) -> z = y or z. Как я это получил? Построил таблицу и получил.
В третьем - таблицы или точно такое же упрощение (через деМоргана), в четвертом тоже, в пятом тупо подставляешь.
>> No.141012 Reply
File: Aa34j8ygYPI.jpg
Jpg, 80.38 KB, 960×720 - Click the image to expand
edit Find source with google Find source with iqdb
Aa34j8ygYPI.jpg
И еще вопрос, помогите разобраться с этим. Я не пойму как делать, что значит наборы? В интернете написано что Булевы функции по разному имеют значения.

Так какая функция, как мне ее брать?
Есть решение, там указано что все наборы где указаны - типа единица. А почему так?
>> No.141015 Reply
File: IMG_20150602_214056.jpg
Jpg, 1934.18 KB, 3264×2448 - Click the image to expand
edit Find source with google Find source with iqdb
IMG_20150602_214056.jpg
>>141010
> Строишь две таблицы истинности, если они одинаковые - ты доказал. Это и называется равносильностью.
Какие две таблицы, я не понял, про какие таблицы речь идет? Вот я сейчас сделал таблицу.
   Алсо что значит указать вид формул? Назвать их или что.
>> No.141016 Reply
>>141015
> что значит указать вид формул
Подозреваю, что в итоге получится какая-нибудь нормальная форма, конъюнктивная или дизъюнктивная. Раз в таблице всего один ноль, то она будет конъюнктивной
> про какие таблицы речь идет?
Про таблицы исходной формулы и той формулы, которую ты получил упрощением.
>> No.141017 Reply
>>141012
Набор - это набор переменных. Пусть есть f(x,y,z) тогда набор (0,1,1) соответствует x=0, y=1, z=1. По другому этот набор можно считать числом в двоичной системе: (0,1,1) = 3.
То есть, тебе дано множество значений переменных, при которых функция определена. Видимо, это частичная функция. Не знаю, как можно построить СДНФ частичной функции, разве только решить, что на остальных наборах она нулевая. Подозреваю, что задание просто плохо сформулировано.
>> No.141019 Reply
File: 9U4Qo-e0IQk.jpg
Jpg, 108.35 KB, 960×720 - Click the image to expand
edit Find source with google Find source with iqdb
9U4Qo-e0IQk.jpg
>>141016
> Про таблицы исходной формулы и той формулы, которую ты получил упрощением.
Спасибо большое! Я понял, строю и сравниваю результаты на T И F

>>141017
Вот тут решение задачи только условие с или и не и наборы слегка другие, но я немного не понял почему единицы. И еще не понял по какому правилу сокращали.
>> No.141021 Reply
>>140868
Пиши на hotnightcode@gmail.com В процессе чтения Геометрии Прасолова и Теоремы Абеля Алексеева. Интересно обсудить некоторые вопросы. Тоже поступаю.
>> No.141022 Reply
File: IMG_20150602_232035.jpg
Jpg, 340.52 KB, 1332×972 - Click the image to expand
edit Find source with google Find source with iqdb
IMG_20150602_232035.jpg
>>141019
Я вот чего не пойму, я правильно де Морганом воспользовался, нужно доказать тождество. Мне кажется я где-то ошибься, ибо не сходиться что-то
>> No.141023 Reply
>>141019
Это решение задачи, в которой f принимает значение 1 на заданных наборах, а на остальных ноль. Про минимизацию довольно неплохо расписано в википедии. Ключевые слова: СДНФ, СКНФ, минимизация.
>> No.141024 Reply
>>141022
Тут всего три переменных, проще таблицу построить. Когда много символов, легко запутаться.
Так, мы подсчитывали, что вероятность правильно перемножить руками две матрицы 4х4 составляет около 50%. (Считая, что мы делаем арифметическую ошибку с вероятностью в 1%)
>> No.141025 Reply
>>141024
Ну я думал про таблицу, но в задании сказано упростить тождество. В целом я верно применил закон или нельзя так?

Блин, эта штука такая интересная, никогда не подумал что языки и исчисления такая сфера, где на основе строится автоматическое доказательство теорем
>> No.141026 Reply
>>141025
Я хотел сказать, что правильно начал? Если да, то тогда доведу до конца упрощение
>> No.141033 Reply
Доброаноны, объясните тему ЖНФ. Почему для нильпотентной матрицы 2х2 её жнф равна
(01
   00) если она не нулевая? И почему если она нулевая, то её ЖНФ равна нулевой матрице 2х2? Помогите пожалуйста, очень нужно)
>> No.141034 Reply
>>141026
Начал правильно, так применять можно. Распространение закона деМоргана на случай нескольких конъюнкций/дизъюнкций легко выводится, тренируйся делать такие выводы сам, не заглядывая никуда, и не задавая вопросов.
>> No.141047 Reply
>>141033
ЖНФ получается, если записать матрицу оператора в специально выбранном "хорошем" базисе. Отсюда следуют ответы на все твои вопросы.
> И почему если она нулевая, то её ЖНФ равна нулевой матрице 2х2?
Нулевая матрица одинаково записывается в любом базисе, в том числе и в нашем "хорошем".
> Почему для нильпотентной матрицы 2х2 её жнф равна
> (01
> 00) если она не нулевая?
На диагонали у ЖНФ стоят собственные числа матрицы, выше диагонали - единицы. У нильпотентной матрицы все собственные числа - нули, иначе она не была б нильпотентной(конечно, я исхожу их того, что мы рассматриваем матрицы над полем, а не над каким-нибудь кольцом с непустым нильрадикалом). Собственно, из этого сразу и получается, что единственный возможный вид ЖНФ для нильпотентной матрицы - такой.
>> No.141058 Reply
>>141047
у любой ненулевой нильпотентной матрицы все собственные значения равны нулю, или только у матрицы размера 2х2?
>> No.141061 Reply
>>141058
А ты подумай. Умножаем A*x = kx, тоже собственный вектор. Умножаем его на A, получаем еще один собственный вектор. Когда мы n раз домножим на A все должно обнулиться. Следовательно k - делитель нуля. Если все это над полем, очевидно k=0.
>> No.141063 Reply
>>141061
>>141047
спасибо большое))
>> No.141064 Reply
>>141063
Ты две скобки забыл открыть.
>> No.141075 Reply
>>136767
Эх, столько задрачивал всю дни матлогику и сегодня на зачет с температурой 37.8 поехал, разнервничался и забыл как расписывается xor, в итоге отл не получил, лол, теперь на следующей неделе приходить надо, задание элементарнейшее попалось, но что-то разноволновался и все вылетело, а в метро без проблем решил, эх.
Сегодня вот попробую другое прорешать и спрошу совета, как только получше будет
>> No.141119 Reply
File: 20150604_151659.jpg
Jpg, 89.60 KB, 640×480 - Click the image to expand
edit Find source with google Find source with iqdb
20150604_151659.jpg
Анон помоги пожалуйста, не могу понять, какой признак использовать чтобы определить , сходится ряд или нет
>> No.141130 Reply
File: 20150604_151659.jpg
Jpg, 89.60 KB, 640×480 - Click the image to expand
edit Find source with google Find source with iqdb
20150604_151659.jpg
помогите пожалуйста определить сходимость или расходимость предела, не могу понять, какой тут нужно использовать признак
>> No.141134 Reply
>>141130
Сходится. ln((n+1)/(n-1)) = ln(1 + 2/(n-1)) при n -> \infty даёт O(1/(n-1)). Так что весь ряд оценивается примерно как сумма n^(-3/2).
>> No.141135 Reply
>>141075
> забыл как расписывается xor
Потому что надо не учить, а понимать. Он расписывается как x~y + y~x потому, что в этом случае выполняется 1+0 = 0+1 = 1 и 1+1=0, легко проверить подстановкой. Два слагаемых нужны для того, чтобы оба случая 1+0 и 0+1 дали одинаковый результат.
>> No.141146 Reply
File: 123213.PNG
Png, 12.12 KB, 694×414 - Click the image to expand
edit Find source with google Find source with iqdb
123213.PNG
>>141135
Ну это понятно, что надо было выводить. Говорю, по пути домой все правильно решил и сверилна вольфраме, сошлось.

Есть еще вопрос.
Как дальше решать? Параллельно читаю андерсона и все равно не понимаю.
Дальше нахожу характеристическое уравнение, а дальше как? (надо бы научиться в ТеХе писать а не в ворде)
>> No.141149 Reply
File: 8YWXi10ZtmU.jpg
Jpg, 49.63 KB, 604×425 - Click the image to expand
edit Find source with google Find source with iqdb
8YWXi10ZtmU.jpg
>>141135
Ты ебанутый, ты че там делаешь? Проще всего его понимать как сложение в Z_2
>> No.141157 Reply
File: 14328480494190.png
Png, 9034.45 KB, 2615×2792 - Click the image to expand
edit Find source with google Find source with iqdb
14328480494190.png
>>141047
тут такой момент. есть обычная матрица 4x4. собственное значение одно ( алг.крат-4, геом. крат - 2). Следовательно ЖФ матрицы будет иметь две клетки, а сумма размерностей этих клеток равна 4, так? Но тогда имеем три варианта - клетка с одним элементом с клеткой 2x2, две клетки 2x2 и клетка 2x2 с единичной клеткой. вот. Эти варианты равнозначны? Если нет, то по какому принципу выбирать?
>> No.141172 Reply
>>141157
Я не очень понимаю, как ты считаешь варианты. Их два - две клетки 2х2 либо клетка 3x3 плюс клетка 1х1. Различить их просто: матрица из двух клеток 2х2 имеет минимальный многочлен второй степени, остальной вариант - третьей.
>> No.141200 Reply
>>141172
многочлен? В смысле характерестический? Он получается четвёртой степени и из него один корень. Или я не в ту сторону смотрю?
>> No.141213 Reply
>>141200
Минимальный многочлен. Он делит характеристический, но, вообще говоря, с характеристическим не совпадает.
Алсо, сделайте новый тред кто-нибудь, этот давно тонет.
>> No.141239 Reply
>>136767
почему выражения не равны?
!((a&&b) || (!a&&!b)) == !(a&&b) && !(!a&&!b)

Почему вольфрам выдает что не равны они? таблица истинности совпадает, деМоргана правильно использовался же, руками несколько раз проверил, фигли мне выдает !=
>> No.141254 Reply
Перекат >>141253


Password:

[ /tv/ /rf/ /vg/ /a/ /b/ /u/ /bo/ /fur/ /to/ /dt/ /cp/ /oe/ /bg/ /ve/ /r/ /mad/ /d/ /mu/ /cr/ /di/ /sw/ /hr/ /wh/ /lor/ /s/ /hau/ /slow/ /gf/ /vn/ /w/ /ma/ /azu/ /wn/ ] [ Main | Settings | Bookmarks | Music Player ]